VisionIAS Research and Analysis February 2024 Modern Indian History (7 Year PYQ Trend Analysis)

You might also like

Download as pdf or txt
Download as pdf or txt
You are on page 1of 86

7 Year PYQ Trend Analysis:

GS PRELIMS (2017 -2023)


MODERN
INDIAN HISTORY
DELHI AHMEDABAD BHOPAL CHANDIGARH GUWAHATI HYDERABAD JAIPUR JODHPUR LUCKNOW PRAYAGRAJ PUNE RANCHI SIKAR
Table of Content
3 A Strategy & Approach - (INDIAN MODERN HISTORY)

4 Questions Source Type Analysis

4 Important Observations

5 Sources to be Referred

6 Advent of Europeans

14 Economic Impact of British Policies

20 British Acts and Policies

38 Swadeshi Movement

44 Gandhian Phase

55 Transfer of Power

58 Peasant Movement and Tribal Rebellions

63 Socio-Religious Movements

69 Working Class Movement

72 Personality Based Questions

80 Chronology

82 Miscellaneous
A STRATEGY & APPROACH - (INDIAN MODERN HISTORY)

UPSC Syllabus
PRELIMS - History of India and Indian National Movement

TOTAL NO. OF QUESTIONS 49

TOPIC WISE QUESTIONS

3 Questions 4 Question
Miscellaneous Advent of Europeans

2 Questions 4 Question
Chronology Economic impact of British Policies

6 Questions
Personality based

2 Questions
Working Class Movement

12 Questions
3 Questions British Acts & Policies
Socio-Religious Movement

3 Questions
Peasant revolts/
Tribal rebellions

1 Questions
Transfer of Power
3 Questions
Swadeshi Movement

6 Questions
Gandhian phase

ONLINE | DELHI | AHMEDABAD | BHOPAL | CHANDIGARH | GUWAHATI | HYDERABAD | JAIPUR | JODHPUR | LUCKNOW | PRAYAGRAJ | PUNE | RANCHI | SIKAR 3
QUESTIONS SOURCE TYPE ANALYSIS
SOURCE TYPE
EM - Essential Material | RM - Reference Material
EN - Essential News | U - Unconventional

41 Question
Essential Material
40
36
32
28
24
20 2 Question
Unconventional
16
3 Question
12 Essential News

8
4

3 Question
Reference Material

IMPORTANT OBSERVATIONS BASED ON


7 YEAR’S PRELIMS QUESTIONS

Mostly the questions in Modern history have come from static and
standard sources which has made the overall difficulty level to
moderate. The facts, however chosen to be tested were difficult.

The topics and themes are almost on the predicted lines wherein
the personality based questions, Indian National movement phase
dominating the pool of the questions asked.

The overall number of questions in the recent years have declined


which is making the preparation and scoring difficult.

There is also seen a link with important events and news in the
questions asked in the modern history section.

ONLINE | DELHI | AHMEDABAD | BHOPAL | CHANDIGARH | GUWAHATI | HYDERABAD | JAIPUR | JODHPUR | LUCKNOW | PRAYAGRAJ | PUNE | RANCHI | SIKAR 4
SOURCES TO BE REFERRED

ESSENTIAL MATERIAL- REFERENCE MATERIAL


NCERTS
Shekhar
Themes in Indian History III Bandopadyay’s Plassey
(New NCERT Class 12th) To Partition
Old NCERT Class 12th (By BL Grover: A New Look At
Bipin Chandra) Modern History
Spectrum’s A Brief History Sumit Sarkar’s Modern
of Modern India India
Bipin Chandra’s India’s
Struggle for Independence

ONLINE | DELHI | AHMEDABAD | BHOPAL | CHANDIGARH | GUWAHATI | HYDERABAD | JAIPUR | JODHPUR | LUCKNOW | PRAYAGRAJ | PUNE | RANCHI | SIKAR 5
Advent of Europeans
Total No. of Questions: 04

Advent of Europeans, Total No.of Questions: 4

2 2
Questions Question

Exchange of British Factories


Commodities

Exchange of Commodities

Year 2018

Q1. The staple commodities of export by the English East Indian Company from
Bengal in the middle of the 18th century were
(a) Raw cotton, oil-seeds and opium
(b) Sugar, salt, zinc and lead
(c) Copper, silver, gold, spices and tea
(d) Cotton, silk, saltpeter and opium

ANSWER D
SOURCE: OLD NCERT Bipin Chandra

EXPLANATION

Indian trade and commerce was expanded in the 17th century due to political stability of the
Mughal empire and Mughal policies that helped in the commercialization of the economy or the
growth of money economy.
During the 17th century India exported the following items to the outside world:
Foodstuffs such as sugar, rice etc.
Indian textiles was in great demand like raw silk.
Indigo
Sugarcane
Cotton
Saltpeter
The best quality of saltpeter was found in Bihar. India exported the saltpetre to the outside world.
Saltpetre supplemented the European sources of gunpowder.
India imported certain metals such as tin and copper as the production of these articles were
insufficient.
Certain spices for food and medicinal purposes, war horses and luxury items such as ivory were
also imported. Hence option (d) is correct answer.
ONLINE | DELHI | AHMEDABAD | BHOPAL | CHANDIGARH | GUWAHATI | HYDERABAD | JAIPUR | JODHPUR | LUCKNOW | PRAYAGRAJ | PUNE | RANCHI | SIKAR 6
Exchange of Commodities

Year 2019

Q2. Which one of the following groups of plants was domesticated in the ‘New
World’ and introduced into the ‘Old World’?
(a) Tobacco, cocoa and rubber
(b) Tobacco, cotton and rubber
(c) Cotton, coffee and sugarcane
(d) Rubber, coffee and wheat

ANSWER A
SOURCE: New NCERT Themes in World History Chapter:- Confrontation of Culture

EXPLANATION

After the voyages of Christopher Columbus in 1492, the Columbian exchange brought New
World crops such as maize, potatoes, sweet potatoes, and manioc to Europe, and Old World
crops such as wheat, barley, rice, and turnips, and livestock including horses, cattle, sheep,
and goats to the America.

Plant of the new World Plant of the old World


(The Americas) (Europe, Africa and Asia)

Beans Avacados Carrot Aimonds


Chill pepper Cacao Eggpiants Apples
Corn Cashews Oats Bananas
Patatoes Papayas Garlic Coffee
Rubber Peanuts Lettuce Grapes
Squash Pineapples Onions Oranges
Sweet patatoes Strawberries Rise Mango
Tamatoes Vanilla Rye Peaches
Tabacco Sugarcane Pears
Tea Pistachios
Wheat Walnuts

ONLINE | DELHI | AHMEDABAD | BHOPAL | CHANDIGARH | GUWAHATI | HYDERABAD | JAIPUR | JODHPUR | LUCKNOW | PRAYAGRAJ | PUNE | RANCHI | SIKAR 7
The Columbian Exchange is the process by which plants, animals, diseases, people, and
ideas have been introduced from Europe, Asia, and Africa to the Americas and vice versa.
It began in the 15th century, when oceanic shipping brought the Western and Eastern
hemispheres into contact.
The exchange began to increase particularly in the wake of Christopher Columbus’s
voyages that began in 1492.
The consequences of the Columbian Exchange profoundly shaped world history.

horses
pigs
potatoes

˖ˢ˥ˡʛˠ˔˜˭˘ʜ

sweet potatoes cattle


chinkens

pumpkins sheep
and squash rats

honeybees
grains
cassava
• wheat
• rice
• barley
tomatoes
• oats
• rye
bananas
ATLANTIC
pineapples OCEAN
olives
sugarcane
avocados The Americas To
Europe, Africa,
EUROPE
NORTH and Asia
N

beans coffee beans


E IA

(kidney, AMERICA citrus fruits


navy,
G B

˟˜ˠ˔ʜ
N M

peaches
A U

peanuts apples and pears


H L
C CO

AFRICA
EX HE

cashews grapes
EUROPE, AFRICA,
T

carrots
cacao AND ASIA
ʛ˙ˢ˥˖˛ˢ˖ˢ˟˔˧˘ʜ TO THE AMERICAS
cabbage
lettuce
vanilla

chili peppers SOUTH onions


AMERICA
PACIFIC soybeans
ATLANTIC
OCEAN weeds
˦˨ˡЄˢ˪˘˥˦ OCEAN • crabgrass
• dandelions
• thistles
• wild oats

tobacco
quinine diseases
• bubonic plage
• smallpox • diphtheria
• typhus • measles
• malaria ϧ˜ˡЄ˨˘ˡ˭˔
• yellow fever • whooping
cough
turkeys

HOW NEW WORLD WAS DISCOVERED?


In 1476 Columbus moved to Lisbon, Portugal, and for many years attempted to gain
support for a journey he was planning to find new trade routes to the Far East. Eventually
Ferdinand and Isabella, the King and Queen of Spain, agreed to finance him.

ONLINE | DELHI | AHMEDABAD | BHOPAL | CHANDIGARH | GUWAHATI | HYDERABAD | JAIPUR | JODHPUR | LUCKNOW | PRAYAGRAJ | PUNE | RANCHI | SIKAR 8
What did Columbus aim to do?
In the 15th and 16th centuries, Europeans wanted to find sea routes to the Far East.
Columbus wanted to find a new route to India, China, Japan and the Spice Islands. If he could
reach these lands, he would be able to bring back rich cargoes of silks and spices. Columbus
knew that the world was round and realised that by sailing west – instead of east around the
coast of Africa, as other explorers at the time were doing – he would still reach his destination. .
After sailing across the Atlantic Ocean for 10 weeks, land was sighted by a sailor called Rodrigo
Bernajo (although Columbus himself took the credit for this). He landed on a small island in the
Bahamas, which he named San Salvador. He claimed the island for the King and Queen of
Spain, although it was already populated.
Columbus called all the people he met in the islands ‘Indians’, because he was sure that he
had reached the Indies. This initial encounter opened up the 'New World' to European
colonisation, which would come to have a devastating impact on indigenous populations.

STUDENTS NOTE
In 2021 and 2022 UPSC asked questions on recurrent
theme - Factories Set Up By Europeans

2021 2022
In the first quarter of seventeenth century, With reference to Indian history, consider
in which of the following was/were the the following statements:
factory/factories of the English East India 1. The Dutch established their
Company located? factories/warehouses on the east coast on
1. Broach lands granted to them by Gajapati rulers.
2. Chicacole 2. Alfonso de Albuquerque captured Goa
3. Trichinopoly from the Bijapur Sultanate.
Select the correct answer using the code 3. The English East India Company
given below. established a factory at Madras on a plot of
land leased from a representative of the
(a) 1 only
Vijayangara empire.
(b) 1 and 2 only
Which of the statements given above are
(c) 3 only correct?
(d) 2 and 3 only (a) 1 and 2 only
(b) 2 and 3 only
(c) 1 and 3 only
(d) 1, 2 and 3

ONLINE | DELHI | AHMEDABAD | BHOPAL | CHANDIGARH | GUWAHATI | HYDERABAD | JAIPUR | JODHPUR | LUCKNOW | PRAYAGRAJ | PUNE | RANCHI | SIKAR 9
British Factories

Year 2021

Q3. In the first quarter of seventeenth century, in which of the following was/were
the factory/factories of the English East India Company located?
1. Broach
2. Chicacole
3. Trichinopoly
Select the correct answer using the code given below.
(a) 1 only
(b) 1 and 2
(c) 3 only
(d) 2 and 3

ANSWER A

SOURCE: Old NCERT Class 12th / Spectrum

EXPLANATION

English East India Company was formed by a group of merchants known as the ‘Merchant
Adventures’ in 1599. A charter to the new Company was granted by Queen Elizabeth (December,
1600) giving it the monopoly of Eastern trade for 15 years.
Establishment of Factories by East India Company:
West Coast: The English established factories at Agra, Ahmadabad, Baroda and Broach by 1619,
all of which were placed under the control of the president and council of the Surat factory. The
com pany acquired Bombay from Charles II on lease at an annual rental often pounds in 1668.
South-eastern Coast: Factories were established at Masulipatam (1611) and Armagaon near
Pulicat (1626).
Eastern India: Factories were set up at Hariharpur and Balasore in Orissa (1633), at Hugli in 1651,
followed by those at Patna, Dacca, Kasimbazar in Bengal and Bihar. Moreover, during Carnatic wars
(18th C), in return for the French help to Salabat Jung, to the throne of Hyderabad as Nizam, Sala
bat granted the French the area in Andhra known as Northern Circars (The Northern Circars were 5
in number: Chicacole (Srikakulam), Rajahmundry, Eluru, Kondapalli and Guntur)-So during the first
quarter of 17th C, Chicacole was under the control of regional kingdoms-there was no East
India company factory here.
Also during the same Carnatic wars , Trichinopoly was under the Kingdom of Arcot, as there was
famous incident of seize of Trichinopoly. There was no EIC factory here too.

ONLINE | DELHI | AHMEDABAD | BHOPAL | CHANDIGARH | GUWAHATI | HYDERABAD | JAIPUR | JODHPUR | LUCKNOW | PRAYAGRAJ | PUNE | RANCHI | SIKAR 10
A PRE-GLIMPSE
EUROPEAN TRADING CENTRES IN INDIA

Agra (E)

Patna

Ahmedabad (E)
Broach (E) Kolkata

Surat (E) Balasore (F)


Diu (P) Daman (P)

Tarapore (P)
BAY OF BANGAL
Mumbai (E)

Yenam (F)
Goa (P) Masulipatnam (D)
Pulicat (D)
Chennai (E)

Pondicherry (F)
Cannanore (P) Tranauebar (Da)
Mahe (F)
Trading entres Calicut (P) Karaikal (F)
Nagapattinam (D)
Da Danish Cochin (P)
D Dutch
E English
SRI LANKA
F French
P Potuguese
INDIAN OCEAN

ONLINE | DELHI | AHMEDABAD | BHOPAL | CHANDIGARH | GUWAHATI | HYDERABAD | JAIPUR | JODHPUR | LUCKNOW | PRAYAGRAJ | PUNE | RANCHI | SIKAR 11
British Factories

Year 2022

Q4. With reference to Indian history, consider the following statements:


1. The Dutch established their factories/warehouses on the east coast on lands granted to them by
Gajapati rulers.
2. Alfonso de Albuquerque captured Goa from the Bijapur Sultanate.
3. The English East India Company established a factory at Madras on a plot of land leased from a
representative of the Vijayangara empire.

Which of the statements given above are correct?


(a) 1 and 2 only
(b) 2 and 3 only
(c) 1 and 3 only
(d) 1, 2 and 3

ANSWER B

SOURCE: Bipin chandra Modern India (Old NCERT)

STUDENTS NOTE
HOW THIS QUESTION COULD BE SOLVED - ELIMINATION OF STATEMENT 1
REQUIRES information on chronology. Hindu Gajapati rulers (c. 1435 –
1541 CE) ruled over Kalinga (Odisha), large parts of Andhra Pradesh
and West Bengal, and the eastern and central parts of Madhya
Pradesh and Jharkhand whereas, The Dutch founded their first factory
in Masaulipatam in Andhra Pradesh in 1605.

EXPLANATION

In 1602, the United East India Company of the Netherlands was formed and given permission by
the Dutch government to trade in the East Indies including India. The Dutch founded their first
factory in Masaulipatam in Andhra Pradesh in 1605. They went on to establish trading centers in
different parts of India and thus became a threat to the Portuguese. They captured Nagapatam
near Madras (Chennai) from the Portuguese and made it their main stronghold in South India.
Hence statement 1 is not correct.

ONLINE | DELHI | AHMEDABAD | BHOPAL | CHANDIGARH | GUWAHATI | HYDERABAD | JAIPUR | JODHPUR | LUCKNOW | PRAYAGRAJ | PUNE | RANCHI | SIKAR 12
Alfonso de Albuquerque served as viceroy of Portuguese India from 1509 to 1515. He was the real
founder of the Portuguese power in the East, a task he completed before his death. He secured for
Portugal the strategic control of the Indian Ocean by establishing bases overlooking all the
entrances to the sea.
Albuquerque acquired Goa from the Sultan of Bijapur in 1510 with ease; the principal port of the
Sultan of Bijapur became “the first bit of Indian territory to be under the Europeans since the time
of Alexander the Great”. Hence statement 2 is correct.
British’s East India Company had entered the then Madras and entered a treaty with the native
chieftains of the land to choose the site of their choice to settle and trade. Company established
a factory in Madras in 1639 on land leased from representatives of Vijayanagara Empire called
the Nayakas.
Originally an uninhabited land, Francis Day and his superior Andrew Cogan of the British East India
Company can be considered the founders of Madras, finally zeroed in on this coastal town and
began construction of St George Fort and houses for their residence on 23 April 1640. Hence
statement 3 is correct.

ONLINE | DELHI | AHMEDABAD | BHOPAL | CHANDIGARH | GUWAHATI | HYDERABAD | JAIPUR | JODHPUR | LUCKNOW | PRAYAGRAJ | PUNE | RANCHI | SIKAR 13
Economic Impact of British Policies

Economic Impact of British Policies, Total No.of Questions: 4

1 3
Questions Question

Land Tenure Economic Impact


Systems of British Rule

Land Tenure Systems

Year 2017

Q.1 Who among the following was/were associated with the introduction of Ryotwari
Settlement in India during the British Rule?
1. Lord Cornwallis
2. Alexander Reed
3. Thomas Munro
Select the correct answer using the code given below:
(a) 1 only
(b) 1 and 3 only
(c) 2 and 3 only
(d) 1, 2 and 3

ANSWER C

SOURCE: Class 8 NCERT - Our Pasts - Ch3 - Ruling The Country Side

EXPLANATION
In the British territories in the south a new system that was devised came to be known as the ryotwar
(or ryotwari ). It was tried on a small scale by Captain Alexander Reed in some of the areas that
weretaken over by the Company after the wars with Tipu Sultan. Subsequently developed by Thomas
Munro, this system was gradually extended all over south India.
Lord Cornwallis was asscoiated with Zamindari/ Permanent Settlement.
Hence option (c) is correct answer.

ONLINE | DELHI | AHMEDABAD | BHOPAL | CHANDIGARH | GUWAHATI | HYDERABAD | JAIPUR | JODHPUR | LUCKNOW | PRAYAGRAJ | PUNE | RANCHI | SIKAR 14
A PRE-GLIMPSE
LAND REVENUE POLICY UNDER BRITISH

BRITISH GOT DIWANI RIGHTS OBJECTIVE WAS TO LAND REVENUE SETTLEMENT


FOR BENGAL, BIHAR AND INCREASE THE LAND SYSTEMS WERE OUT
ORISSA IN 1765. REVENUE COLLECTION. PRODUCT OF THE SAME.

PERMANENT RYOTWARI SYSTEM MAHALWARI


SETTLEMENT SYSTEM (1820) SYSTEM (1819)
(1793)

MAHALWARI
SYSTEM:
Gangetic valley,
north-west provinces, PERMANENT
parts of central India SETTLEMENT
& punjab. SYSTEM:
Bengal, Bihar,
Orissa, etc

RYOTWARI SYSTEM:
Malabar,
Coimbatore,
Madras and
Madurai

REVENUE PERMANENT RYOTWARI MAHALWA


SYSTEM SETTLEMENT SYSTEM RI SYSTEM

Lord Cornwallis (In 1793) 1792: Alexander Reed in Holt Mackenzie in 1819.
Baramahal.
1801-1807: Sir Thomas
Munro continued.
1820: Munro returned to
Introduced By India and reintroduction.

Bengal, Bihar, Orissa, Malabar, Coimbatore, Gangetic valley, north-west


Banaras division of modem Madras and Madurai, Later provinces, parts of central
UP, and Northem Carnatic. extended to Maharashtra, India and Punjab. Nearly in
Nearly 19% of British territory East Bengal, parts of Assam 30% area of British India.
Region in India. and Coorg.

ONLINE | DELHI | AHMEDABAD | BHOPAL | CHANDIGARH | GUWAHATI | HYDERABAD | JAIPUR | JODHPUR | LUCKNOW | PRAYAGRAJ | PUNE | RANCHI | SIKAR 15
Economic Impact of British Rule

Year 2018

Q.2 Economically, one of the results of the British rule in India in the 19th century was the
(a) increase in the export of Indian handicrafts
(b) growth in the number of Indian owned factories
(c) commercialization of Indian agriculture
(d) rapid inrease in the urban population

ANSWER C

SOURCE: Old NCERT Ch-8 Economic Impact of British Rule in India

EXPLANATION

Economic Impact of British rule-


Deindustrialisation - Ruin of artisans and handicraftsmen: cheap and machine made goods
imports flooded the Indian market after the Charter Act of 1813 allowing one-way free trade for
British citizens. On the other hand, Indian products found it more and more difficult to penetrate
the European markets. After 1820, European markets were virtually closed to Indian exports. Even
the newly introduced rail network helped the European products to reach the remotest corners of the
country. Hence, option (a) is not correct.
Impoverishment of Peasantry - The Govt. was only interested in maximisation of rents and in
securing its share of revenue , had enforced the Permanent Settlement in large parts. Transferability
of land was one feature of the new settlement which caused great insecurity to the tenants who lost
all their traditional rights in land.
Development of Modern Industries - It was only in the second half of 19th Century that modern
machine based industries started coming up in India. The first cotton textile mill was started in 1853
in Bombay by Cowasjee Nanabhoy and the first jute mill came up in 1855 in Rishra (Bengal). But most
of the industries were foreign owned and controlled by British managing agencies. Hence, option
(b) is not correct.
Commercialisation of Indian Agriculture - In the latter half of the 19th century, another significant
trend was the emergence of the commercialisation of agriculture. Agriculture began to be
influenced by commercial considerations. Certain specialised crops began to be grown not for
consumption in the village but for sale in the national and even international markets. Commercial
crops like cotton, jute, groundnut, oilseeds, sugarcane, tobacco, etc. were more remunerative than
foodgrains. Hence option (c) is the correct answer.
Another feature of deindustrialisation was the decline of many cities and a process of ruralisation
of India. Many artisans, faced with diminishing returns and repressive policies of the Company
abandoned their professions and moved to villages and took to agriculture. This resulted in
increased pressure on land. An overburdened agriculture sector was a major cause of poverty
overburdened agriculture sector was a major cause of poverty during British rule and this upset the
village economic set-up. Hence, option (d) is not correct.

ONLINE | DELHI | AHMEDABAD | BHOPAL | CHANDIGARH | GUWAHATI | HYDERABAD | JAIPUR | JODHPUR | LUCKNOW | PRAYAGRAJ | PUNE | RANCHI | SIKAR 16
Economic Impact of British Rule

Year 2020

Q.3 Indigo cultivation in India declined by the beginning of the 20th century because of
(a) peasant resistance to the oppressive conduct of planters
(b) its unprofitability in the world market because of new inventions
(c) national leaders' opposition to the cultivation of indigo
(d) Government control over the planters

ANSWER B

SOURCE: Our Past-III, Ncert class -8th, chapter: RULING THE COUNTRYSIDE
FROM PLASSEY TO PARTITION by Sekhar Bandyopadhyaya.
Old NCERT Class 12th by Bipin Chandra: Gandhiji's Early Career and Activism.

EXPLANATION

Economic Impact of British rule-


Indigo, the blue dye, was extracted from plants in ancient times, some 5000-6000 years ago
(3000-4000 BCE), both in the Old (Asia, Africa and Europe) and New (Americas) Worlds.
It got its name Indigo, because it reached Europe from Indus Valley, India and later from other parts
of India by the Portuguese and other European sailors.
It was commercially encouraged and traded by the British, firstly by the cultivation of indigo plant
and production of the dye in South Carolina, USA in mid-18th century, which was then a British colony.
However, this stopped after the British colonies in USA gained their freedom after American
Revolutionary War (1775-1783).
It was then that British East India Company (EIC) started its production in Bengal and part of the
current Bihar states of India and continued it until the second decade of 20th century. The Company
looked for ways to expand the area under indigo cultivation in India. From the last decades of the
eighteenth century, indigo cultivation in Bengal rapidly expanded. Only about 30% of indigo
imported to Britain in 1788 was from India. This figure went up to 95% by 1810.
Commercial agents and officials of the Company began investing in indigo production to increase
their profit. Many Company officials even left their jobs to look after their indigo business. Many
people from Scotland and England came to India and became planters; to grab the opportunity.
The Company and banks were giving loans for indigo cultivation at that time.
The development of synthetic indigo in the mid-19th century marked a significant shift in the indigo
industry. The English businesses relying on natural indigo faced challenges as the synthetic
alternative gained prominence. The decline in both quantity and value of natural indigo during the
late 19th and early 20th centuries further intensified the tough market conditions for traditional
indigo enterprises. Hence option (b) is the correct answer.

ONLINE | DELHI | AHMEDABAD | BHOPAL | CHANDIGARH | GUWAHATI | HYDERABAD | JAIPUR | JODHPUR | LUCKNOW | PRAYAGRAJ | PUNE | RANCHI | SIKAR 17
Indigo Movement

European Indigo planters in Bengal forced pesants to grow indigo instead of more
Preceding profitable crops like rice.
Events Peasants were intimidated through kidnappings, flogging, attacks on women and
children, demolition of houses and crops.

Key Leaders like Digamber Biswas and Bishnu Biswas organised a counter force against
Features of the planters' attacks.
the Pesants resorted to rent strike and initiated legal action with the help of Bengal
Movement Intellgensia.

An indigo commission was appointed. Based on the reports, the government notified
Consequ- that the pesants could not be compelled to grow indigo.
ences Planters closed down the factories and indigo cultivation was virtually wiped out
from Bengal by 1860.

In this system, the planter produced indigo on those lands which were under his
Nij Indigo
direct control. The planter either bought the land or rented it from other zamindars. He
Cultivation
directly employed labourers to produce indigo.

Under the ryoti system, indigo cultivation was done by the ryots. The planters made
the ryots to sign a contract or an agreement (satta).
Sometimes, they pressurized the village headmen to sign the contract on behalf of the
Ryoti ryots.
Indigo
After signing the contract, the ryots got cash advances from the planters. But after
Cultivation taking the loan, the ryot was committed to grow indigo on at least 25% of his land
holding.
Seeds and drills were provided by the planter. The cultivators prepared the soil, sowed
the seed and looked after the crop.

Economic Impact of British Rule

YEAR 2020

Q4.Which of the following statements correctly explain the impact of Industrial


Revolution on India during the first half of the nineteenth century?
(a) Indian handicrafts were ruined.
(b) Machines were introduced in the Indian textile industry in large numbers.
(c) Railway lines were laid in many parts of the country
(d) Heavy duties were imposed on the imports of British manufactures "

ANSWER A

SOURCE: Bipin Chandra’s India’s Struggle For Independence.

ONLINE | DELHI | AHMEDABAD | BHOPAL | CHANDIGARH | GUWAHATI | HYDERABAD | JAIPUR | JODHPUR | LUCKNOW | PRAYAGRAJ | PUNE | RANCHI | SIKAR 18
EXPLANATION

India was a major player in the world export market for textiles in the early 18th century, but by the
middle of the 19th century it had lost all of its export market and much of its domestic market.
At the begining of Industral revolution cotton industries developed in England, industrial groups
began worrying about imports from other countries. They pressurised the government to impose
import duties on cotton textiles so that Manchester goods could sell in Britain without facing any
competition from outside. At the same time industrialists persuaded the East India Company to
sell British manufactures in Indian markets as well. Exports of British cotton goods increased
dramatically in the early nineteenth century.
Cotton weavers in India thus faced two problems at the same time: their export market collapsed,
and the local market shrank, being glutted with Manchester imports. Produced by machines at
lower costs, the imported cotton goods were so cheap that weavers could not easily compete
with them. By the 1850s, reports from most weaving regions of India narrated stories of decline
and desolation. Thus, Indian handicrafts were ruined extensively. Hence option (a) is the correct
answer.

ONLINE | DELHI | AHMEDABAD | BHOPAL | CHANDIGARH | GUWAHATI | HYDERABAD | JAIPUR | JODHPUR | LUCKNOW | PRAYAGRAJ | PUNE | RANCHI | SIKAR 19
British Acts & Policies
Regulation Act, Charter Acts, GOI Acts etc

British Acts & Policies, Total No.of Questions: 12

6 3 3
Questions Question Questions

Regulation Act, Charter Administrative Education


Acts, GOI Acts etc Policy

STUDENTS NOTE
Solvability - In Act based questions usually UPSC sets traps
by inter-changing the provisions with some other Acts and
policies.

British Acts and Policies

Year 2017

Q1. In the context of Indian history, the principle of ‘Dyarchy (diarchy)’ refers to

(a) Division of the central legislature into two houses.


(b) Introductions of double government i.e., Central and State governments.
(c) Having two sets of rulers; one in London and another in Delhi.
(d) Division of the subjects delegated to the provinces into two categories.

ANSWER D
SOURCE: Spectrum A Brief History Of Modern India
Old NCERT, Class XII, Modern India

EXPLANATION

What is Dyarchy?
Under the GOI Act, 1919, the Provincial Legislative Councils were enlarged and the majority of their
members were to be elected. Thus, was introduced dyarchy for the executive at the level of the
provincial government.
The Dyarchy system provided more powers to the Provincial governments. Under this system
some subjects, such as Land revenue administration, famine relief, irrigation, administration of
justice, law and order, newspapers, borrowing, forests etc., were called 'reserved' subjects and
remained under the direct control of the Governor; others such as education, public health &
sanitation, public works, agriculture, fisheries, religious endowments, local self governments,
medical services etc, were called 'transferred' subjects and were to be controlled by ministers
responsible to the legislatures. Hence option (d) is the correct answer.
ONLINE | DELHI | AHMEDABAD | BHOPAL | CHANDIGARH | GUWAHATI | HYDERABAD | JAIPUR | JODHPUR | LUCKNOW | PRAYAGRAJ | PUNE | RANCHI | SIKAR 20
British Acts and Policies

YEAR 2018

Q2. In the Federation established by The Government of India Act of 1935, residuary
powers were given to the
(a) Federal Legislature
(b) Governor General
(c) Provincial Legislature
(d) Provincial Governors

ANSWER B

SOURCE: Spectrum A Brief History Of Modern India

EXPLANATION
The Government of India Act of 1935 envisaged:
An all-India federation consisting of eleven Governor’s provinces, six Chief Commissioner’s
provinces, and such Indian States as would agree to join the federation.
The subjects were divided into three Lists i.e. Federal, Provincial and Concurrent.
The provincial legislatures were given exclusive power to legislate with respect to matters in the
Provicial list.
The federal legislature had the exclusive power to make law on matters in the Federal List. The
federal and the provincial legislatures had concurrent jurisdiction with respect to matters in the
Concurrent List.
In case of conflict between a provincial law and a federal law on a matter enumerated in the
Concurrent List, the latter was to prevail, and the former would, to the extent of the repugnancy be
void.
Residuary powers were vested in the Governor-General, who could, in his discretion, assign any
such power by a public notification to the federal legislature or the provincial legislature. Hence
option (b) is the correct answer.

British Acts and Policies

Year 2019

Q3. Consider the following statements about ‘the Charter Act of 1813’:
1. It ended the trade monopoly of the East India Company in India except for trade in tea and
trade with China.
2. It asserted the sovereignty of the British Crown over the Indian territories held by the Company.
3. The revenues of India were now controlled by the British Parliament.
Which of the statements given above are correct?

(a) 1 and 2 only


(b) 2 and 3 only
(c) 1 and 3 only
(d) 1, 2 and 3 only

ANSWER A
SOURCE: Spectrum A Brief History Of Modern India

ONLINE | DELHI | AHMEDABAD | BHOPAL | CHANDIGARH | GUWAHATI | HYDERABAD | JAIPUR | JODHPUR | LUCKNOW | PRAYAGRAJ | PUNE | RANCHI | SIKAR 21
EXPLANATION
Main Provisions of Charter Act, 1813
Monopoly of East India Company to trade with India was brought to an end but the company
retained the trade with China and the trade in tea. Hence statement 1 is correct.
The company was allowed to continued territorial possession for period of 20 years i.e. from 1833 to
1853, without prejudice to the sovereignty of the Crown. The company was held in trust to the Crown.
Hence statement 2 is correct.
The Act required the Company to maintain its territorial and commercial accounts separately.
Also,the Company was put under greater control of the British Crown, but could retain its hold on the
details of administration and Indian revenues. From 1853 ,the revenues of India were controlled by
the British Parliament. Hence statement 3 is not correct.

Additional Information: Charter Act of 1813


Background – In the British Parliament, discussion on the privileges enjoyed exclusively
by East India Company vis-à-vis other English companies in the wake of laissez faire
and introduction of Continental System by Napolean which had closed European ports
to British trade.
East India Company was thus deprived of its trade monopoly with India but still
enjoyed trade monopoly with China and trade in tea.
Constitutional position of British territories in India were defined for the first time –
separate accounts for commercial transactions and territiorial revenues.
It enlarged the powers of Board of Control created under Pitts India Act, 1784.
Provinding a sum of 1 lakh rupees annually for the revival and improvement of
literature, promotion of sciences.

British Acts and Policies

YEAR 2021

Q4. Consider the following statements:


1. The Montagu-Chelmsford Reforms of 1919 recommended granting voting rights to
all the women above the age of 21.
2. The Government of India Act of 1935 gave women reserved seats in legislature.
Which of the statements given above is/are correct?
(a) 1 only
(b) 2 only
(c) Both 1 and 2
(d) Neither 1 nor 2

ANSWER B

SOURCE: Spectrum A Brief History Of Modern India

ONLINE | DELHI | AHMEDABAD | BHOPAL | CHANDIGARH | GUWAHATI | HYDERABAD | JAIPUR | JODHPUR | LUCKNOW | PRAYAGRAJ | PUNE | RANCHI | SIKAR 22
EXPLANATION

In line with the government policy contained in Montagu’s statement of August 1917, the government
announced further constitutional reforms in July 1918, known as Montagu-Chelmsford reforms,
Based on these, the Government of India Act, 1919 was enacted. Under the Act, Women were given
the right to vote. But there was no universal adult suffrage. Hence statement 1 is not correct.
The Government of India Act, 1935 extended the principle of communal representation by providing
separate electorates for depressed classes (scheduled castes), women and labourers (workers). A
separate electorate meant that not only were the seats reserved for a specific group, but voting for
the reserved constituency was allowed for only members of that specific community. Hence
statement 2 is correct.

Additional Information: GoI Act 1935


The outcome of the Third Round Table conference was the 'White Paper" issued by the
Government. On the basis of this paper, the Government of India Act 1935 was to be passed.
Round Table Conferences (1930-32) were a series of meetings in three sessions called by the
British government to consider the future constitution of India. The conference resulted from a
review of the Government of India Act of 1919, undertaken in 1927 by the Simon Commission,
whose report was published in 1930. The conference was held in London.
It introduced bicameralism in six out of eleven provinces. Thus, the legislatures of Bengal,
Bombay, Madras, Bihar, Assam and the United Provinces were made bicameral consisting of a
legislative council (upper house) and a legislative assembly (lower house). However, many
restrictions were placed on them.
It further extended the principle of communal representation by providing separate electorates
for depressed classes (scheduled castes), women and labour (workers).
It abolished the Council of India, established by the Government of India Act of 1858. The
secretary of state for India was provided with a team of advisors.
It extended franchise. About 10 per cent of the total population got the voting right.
It provided for the establishment of a Reserve Bank of India to control the currency and credit
of the country.
It provided for the establishment of not only a Federal Public Service Commission but also a
Provincial Public Service Commission and Joint Public Service Commission for two or more
provinces.
It provided for the establishment of a Federal Court, which was set up in 1937.

ADMISSION OPEN

ONLINE | DELHI | AHMEDABAD | BHOPAL | CHANDIGARH | GUWAHATI | HYDERABAD | JAIPUR | JODHPUR | LUCKNOW | PRAYAGRAJ | PUNE | RANCHI | SIKAR 23
British Acts and Policies

Year 2022

Q5. In the Government of India Act 1919, the functions of Provincial Government were
divided into "Reserved and Transferred" subjects. Which of the following were treated
as "Reserved" subjects?
1. Administration of Justice
2. Local Self-Government
3. Land Revenue
4. Police
Select the correct answer using the code given below:
(a) 1, 2 and 3
(b) 2, 3 and 4
(c) 1, 3 and 4
(d) 1, 2 and 4

ANSWER C
SOURCE: Spectrum’s A Brief History Of Modern India

STUDENTS NOTE
Elimination Logic - British Policies and Acts had centralising
tendencies, important subjects like Law, Police, Revenue etc
were kept under directly under them whereas subjects which
involved local subjects like Local Self Government were
transferred to the Indians.

EXPLANATION

About GOI 1919 Act


Introduction of Diarchy at the provinces ie. classification of the central and provincial
subjects. The provincial subjects were divided into two groups: One was reserved subjects
(essential areas of law enforcement such as justice, police, revenue) were kept with the
Governor and transferred subjects (public health, public works, education etc.) were kept
with the Indian Ministers. This division of subjects was basically what they meant by
introducing the Diarchy.

It introduced, for the first time, bicameralism and direct elections in the country. Thus, the
Indian Legislative Council was replaced by a bicameral legislature consisting of an Upper
House (Council of State) and a Lower House (Legislative Assembly). The majority of members of
both the Houses were chosen by direct election.
It required that the three of the six members of the Viceroy's executive Council (other than
the commander-in-chief) were to be Indian.
It extended the principle of communal representation by providing separate electorates for
Sikhs, Indian Christians, Anglo-Indians and Europeans.
It granted franchise to a limited number of people on the basis of property, tax or education.
It created a new office of the High Commissioner for India in London and transferred to him
some of the functions hitherto performed by the Secretary of State for India.

ONLINE | DELHI | AHMEDABAD | BHOPAL | CHANDIGARH | GUWAHATI | HYDERABAD | JAIPUR | JODHPUR | LUCKNOW | PRAYAGRAJ | PUNE | RANCHI | SIKAR 24
It provided for the establishment of a public service commission, Hence, a Central Public Service
Commission was set up in 1926 for recruiting civil servants.
It separated, for the first time, provincial budgets from the Central budget and authorised the
provincial legislatures to enact their budgets.
It provided for the appointment of a statutory commission to inquire into and report on its working
after ten years of its coming into force. Hence option (c) is the correct answer.

British Acts and Policies

Year 2023

Q6. By which one of the following Acts was the Governor General of Bengal designated
as the Governor General of India?
(a) The Regulating Act
(b) The Pitt's India Act
(c) The Charter Act of 1793
(d) The Charter Act of 1833

ANSWER D

SOURCE: Spectrum A Brief History Of Modern India

EXPLANATION
Charter Act of 1833: This Act was the final step towards centralisation in British India.
It made the Governor-General of Bengal as the Governor- General of India and vested in
him all civil and military powers.
The act created, for the first time, the Government of India having authority over the entire
territorial area possessed by the British in India.
Lord William Bentick was the first Governor-General of India.
It deprived the Governor of Bombay and Madras of their legislative powers. The
Governor-General of India was given exclusive legislative powers for the entire British India.
Hence option (d) is the correct answer.

ADMISSION OPEN

ONLINE | DELHI | AHMEDABAD | BHOPAL | CHANDIGARH | GUWAHATI | HYDERABAD | JAIPUR | JODHPUR | LUCKNOW | PRAYAGRAJ | PUNE | RANCHI | SIKAR 25
A PRE-GLIMPSE
LEGISLATIVE ACTS BY THE
BRITISH

Regulating Act, 1773


Recognized, for the first time, the political and administrative functions of the company
apart from trading.
Governor of Bengal became the Governor General of Bengal (GGOB)
Warren Hastings was first Governor General of Bengal
4 members Executive Council was formed to assist the Governor General of Bengal
Established Supreme Court at Calcutta
Private trade was prohibited for the servants of the Company
Governor General in council were also required to be appointed for Madras and Bombay
but they were to act in subordination to the governor general in Calcutta.

Pitt’s India Act, 1784


Board of control and Court of Directors was set up to manage the political affairs and
commercial affairs respectively.
It distinguished between commercial and political functions of company.
Board of control was empowered to manage military, civil and revenue matters
Company's territories were called as British Possessions in India
The Supreme Court of Calcutta was meant only for english subjects.

Charter Act, 1813


Company was deprived of its trade monopoly with India but still enjoyed trade monopoly
with China and trade in tea.
Constitutional position of British territories in India were defined for the first time
Provinding a sum of 1 lakh rupees annually for the revival and improvement of literature and
sciences.

Charter Act, 1833


It was the Final step towards centralization and EIC became purely an administrative body
It completely ended trade monopoly of EIC
Made Governor General of Bengal as the Governor General of India (GGol)
All the military and civil powers were vested in him
Lord William Bentick was 1st GGol
Bombay and Madras presidencies lost legislative powers
It enlarged the Executive council by the addition of fourth member (Law Member)

Charter Act, 1853


It separated, for the first time, the legislative and executive functions of the Governor-General's
Council.
6 members legislative council was formed. 4 out of 6 legislative members were appointed by the
provincial governments of Madras, Bombay, Bengal and Agra.
Civil service was thrown open to Indians through open competition
The Act for the first time introduced local representation in the Indian (Central) Legislative Council.

ONLINE | DELHI | AHMEDABAD | BHOPAL | CHANDIGARH | GUWAHATI | HYDERABAD | JAIPUR | JODHPUR | LUCKNOW | PRAYAGRAJ | PUNE | RANCHI | SIKAR 26
Government of India Act, 1858
Abolished the East India Company, and transferred the powers of government, territories and
revenues to the British Crown.
Changed the designation of the Governor-General of India to that of Viceroy of India.
Lord Canning became the 1st Viceroy
The office of secretary of state who was a member of British Cabinet was established
The Crown will govern India directly through a Secretary of State for India, who was to exercise the
powers which were being enjoyed by the Court of Directors and Board of control.
A 15-member Council of India was established to assist the secretary of state for India.
The council was an advisory body. The secretary of state was made the chairman of the council.

Indian Councils Act, 1861


Beginning of representative institutions by involving Indians in the lawmaking process.
Viceroy could nominate some Indians as non official members of his expanded council.
Initiated the process of Decentralization, Legislative powers of Bombay and Madras
presidencies were restored.
New Legislative Councils were set up in Bengal, NWFP and Punjab.
Viceroy was empowered to make rules and orders for the convenient functioning of business
and transaction of the council
Portfolio system was introduced which laid the foundations of cabinet government in India
Viceroy could issue ordinances without the concurrence of Legislative Council(LC) during
emergency, the limit for such ordinance was set as 6 months
Final passing of the bill needed viceroy’s approval. Even if approved by the viceroy, the
secretary of state could disallow legislation.

Indian Councils Act, 1892


The Indian Legislative Council was enlarged.
The universities, district boards, municipalities, zamindars, trade bodies were empowered to
recommend members to the provincial councils.
Although the term ‘election’ was firmly avoided in the act, an element of indirect election was
accepted
Members of the legislatures were now entitled to express their views upon financial statements.
1909:
The strength of the Imperial Legislative Council was increased but official majority was retained
in Imperial LC.
The number of members of the Provincial LC was increased and non-official majority in PLC was
allowed .
The powers of the legislative councils, both central and provincial, were increased.
Introduced a system of communal representation by awarding separate Electorate for Muslims.

Morley - Minto Reforms, 1909


The strength of the Imperial Legislative Council was increased but official majority was retained in
Imperial LC.
The number of members of the Provincial LC was increased and non-official majority in PLC was
allowed.
Members were now allowed to ask supplementary questions, move resolution on budgets etc
ONLINE | DELHI | AHMEDABAD | BHOPAL | CHANDIGARH | GUWAHATI | HYDERABAD | JAIPUR | JODHPUR | LUCKNOW | PRAYAGRAJ | PUNE | RANCHI | SIKAR 27
Indians were now allowed to become members of Viceroy's Executive Council
Satyendra Sinha was first Indian member, he was the Law member
Introduced the system of communal representation by accepting separate electorate for
Muslims
Minto was called the father of communal electorate

Montagu Chelmsford Reforms, 1919


The act introduced dyarchy in the provinces, which divided provincial subjects as transferred
and reserved subjects.
Transferred subjects were to be administered by the governor with the aid of ministers
responsible to the LC
Reserved subjects to be administered by the governor and his executive council without being
responsible to the LC Diarchy at the provincial level
Bicameralism was introduced in the Central Legislative Assembly.
The act separated for the first time the provincial and central budgets.
A High Commissioner for India was appointed, who was to hold his office in London for six years.
The Secretary of State now to be paid by the British Exchequer.
Direct elections were introduced and Limited Franchise was awarded on the basis of property,
tax & education.
3 out of 6 members of the EC were to be Indians
The principle of communal representation was extended with separate electorates for Sikhs,
Christians, and Anglo-Indians, Muslims

Government of India ACT, 1935


Provided for an All India Federation Introduced bicameralism in provinces
1. Provinces Separate electorates for the depressed classes
2. Princely states Abolished Council of India and SSI was provided
Federal, provincial and concurrent lists were with a team of advisors
made Extended franchise further about 10 per cent of
Federation never came into existence as the total population getting the right to vote.
princely states denied Provided for setting up RBI to control currency
Dyarchy in the provinces was abolished and and credit
provinces were given autonomy Provided for establishing federal PSC, Provincial
Provided for dyarchy at the centre, but did not PSCs and Joint PSCs
come into effect at all Provided for the establishment of a Federal
There was a provision for joint sitting in cases Court
of deadlock between the houses
Residuary legislative powers were subject to
the discretion of the governor general

Indian Independence Act, 1947


End of British rule and declared India independent and sovereign
Provided for two dominion states : India and Pakistan
Boundaries between the two dominion states were to be determined by a Boundary
Commission which was headed by Sir Cyril Radcliff
Provided for partition of Punjab & Bengal and separate boundary commissions to demarcate
the boundaries between them.

ONLINE | DELHI | AHMEDABAD | BHOPAL | CHANDIGARH | GUWAHATI | HYDERABAD | JAIPUR | JODHPUR | LUCKNOW | PRAYAGRAJ | PUNE | RANCHI | SIKAR 28
STUDENTS NOTE
In 2018 itself UPSC asked three questions from Education
related policies of british government.

2018
Which of the following led to the (a) 1 and 2 only
introduction of English Education in India? (b) 2 and 3 only
1. Charter Act of 1813 (c) 1 and 3 only
2. General Committee of Public Instruction, (d) 1, 2 and 3
1823
3. Orientalist and Anglicist Controversy.
2018
Select the correct answer using the code
given below: With reference to educational institutions
during colonial rule in India, consider the
(a) 1 and 2 only
following pairs:
(b) 2 only
Institution Founder
(c) 1 and 3 only
1. Sanskrit College at Benaras - William
(d) 1, 2 and 3
Jones

2018 2. Calcutta Madarsa - Warren


Hastings
Regarding Wood's Dispatch, which of the 3. Fort William College - Arthur
following statements are true? Wellesley
1. Grants-in-Aid system was introduced. Which of the pairs given above is/are
2. Establishment of universities was correctly matched?
recommended. (a) 1 and 2 only
3. English as a medium of instruction at all (b) 2 only
levels of education was recommended. (c) 1 and 3 only
Select the correct answer using the code (d) 3 only
given below:

Education Based Questions

YEAR 2018

Q7. Which of the following led to the introduction of English Education in India?
1. Charter Act of 1813
2. General Committee of Public Instruction, 1823
3. Orientalist and Anglicist Controversy
Select the correct answer using the code given below:
(a) 1 and 2 only
(b) 2 only
(c) 1 and 3 only
(d) 1, 2 and 3

ANSWER D
SOURCE: Spectrum A Brief History Of Modern India

ONLINE | DELHI | AHMEDABAD | BHOPAL | CHANDIGARH | GUWAHATI | HYDERABAD | JAIPUR | JODHPUR | LUCKNOW | PRAYAGRAJ | PUNE | RANCHI | SIKAR 29
EXPLANATION
CHARTER ACT OF 1813: The Act incorporated the principle of encouraging learned Indians and
promoting knowledge of modern sciences in the country.
The Act directed the Company to sanction one lakh rupees annually for this purpose. However,
even this petty amount was not made available till 1823, mainly because of the controversy raged
on the question of the direction that this expenditure should take.
In 1823, the Governor-General-in Council appointed a “General Committee of Public Instruction”,
which had the responsibility to grant the one lakh of rupees for education. That committee
consisted of ten European members of which Lord Macaulay was the president. The committee
decided to spend major portions from the grant for the improvement of oriental literature.
Orientalist-Anglicist Controversy: Within the General Committee on Public Instruction, the
Anglicists argued that the government spending on education should be exclusively for modern
studies.
The Orientalists said while western sciences and literature should be taught to prepare students
to take up jobs, emphasis should be placed on expansion of traditional Indian learning.
Even the Anglicists were divided over the question of medium of instruction—one faction was
for English language as the medium, while the other faction was for Indian languages
(vernaculars) for the purpose.
Lord Macaulay's Minute (1835), settled the row in favour of Anglicists—the limited government
resources were to be devoted to teaching of western sciences and literature through the
medium of English language alone. Hence option (d) is the correct answer.

Education Based Questions

YEAR 2018

Q8. Regarding Wood's Dispatch, which of the following statements are true?
1. Grants-in-Aid system was introduced.
2. Establishment of universities was recommended.
3. English as a medium of instruction at all levels of education was recommended.
Select the correct answer using the code given below:
(a) 1 and 2 only
(b) 2 and 3 only
(c) 1 and 3 only
(d) 1, 2 and 3

ANSWER A

SOURCE: Spectrum A Brief History Of Modern India

EXPLANATION

In 1854, Charles Wood prepared a despatch on an educational system for India. Considered the
Hence "Magna Carta of English Education in. India", this document was the first comprehensive plan
for the spread of education in India.
It asked the Government of India to assume responsibility for education of the masses, thus
repudiating the 'downward filtration theory', at least on paper.
It systematised the hierarchy from vernacular primary schools in villages at bottom, followed by

ONLINE | DELHI | AHMEDABAD | BHOPAL | CHANDIGARH | GUWAHATI | HYDERABAD | JAIPUR | JODHPUR | LUCKNOW | PRAYAGRAJ | PUNE | RANCHI | SIKAR 30
Anglo-Vernacular High Schools and an affiliated college at the district level, and affiliating
universities in the presidency towns of Calcutta, Bombay and Madras.
It recommended English as the medium of instruction for higher studies and vernaculars at
school level. Hence only statement 3 is not correct
It laid stress on female and vocational education, and on teachers’ training.
It laid down that the education imparted in government institutions should be secular.
It recommended a system of grants- in-aid to encourage private enterprise.
The Despatch recommended the establishment of universities in the three Presidency towns of
Calcutta, Bombay and Madras.
The universities were to be modeled after the London University and these were to have a senate
comprising of a Chancellor, a Vice-Chancellor, and fellows who were nominated by the
Government.
The Universities would confer degrees to the successful candidates after passing the
examinations, (of Science or Arts Streams) conducted by the Senate.
The universities were to organize departments not only of English but also of Arabic, Sanskrit and
Persian, as well as law and civil engineering. Hence option(a) is the correct answer.

Education Based Questions

YEAR 2018

Q9. With reference to educational institutions during colonial rule in India, consider the
following pairs:
Institution Founder
1. Sanskrit College at Benaras - William Jones
2. Calcutta Madarsa - Warren Hastings
3. Fort William College - Arthur Wellesley
Which of the pairs given above is/are correctly mached ?
(a) 1 and 2only
(b) 2 only
(c) 1 and 3 only
(d) 3 only

ANSWER B

SOURCE: Spectrum A Brief History Of Modern India

EXPLANATION

Pair 1 is not correctly matched: In 1791, Jonathan Duncan started the Sanskrit College at Benares.
Pair 2 is correctly matched: Calcutta Madrasa was established by Warren Hastings in 1781 for the
study of Muslim law and related subjects.
Pair 3 is not correctly matched: Fort William College was set up by Lord Richard Wellesley in 1800 for
training of civil servants of the Company in languages and customs of Indians.

ONLINE | DELHI | AHMEDABAD | BHOPAL | CHANDIGARH | GUWAHATI | HYDERABAD | JAIPUR | JODHPUR | LUCKNOW | PRAYAGRAJ | PUNE | RANCHI | SIKAR 31
A PRE-GLIMPSE
DEVELOPMENT OF EDUCATION

1781
The Calcutta Madrasa was established by
Warren Hastings.

The Sanskrit College was established


by Jonathan Duncan, the resident, at
Benaras. 1791

1800
The Fort Williams college was established by
Lord Richard Wellesley.

Charter Act of 1813: Directed the

1813
Company to sanction one lakh rupees
annually for encouraging leared Indians
and promote modem sciences.

ONLINE | DELHI | AHMEDABAD | BHOPAL | CHANDIGARH | GUWAHATI | HYDERABAD | JAIPUR | JODHPUR | LUCKNOW | PRAYAGRAJ | PUNE | RANCHI | SIKAR 32
Macaulay minute
Settled the row in favour of Anglicists - teaching

1835 of Western sciences and literature through the


medium of English language alone.
Neglected mass education (proposed
downward filtration theory).

Woods Despatch
"Magna Carta of English Education in India"
Asked government of India to assume
responsibility for education of the masses
( Repudiated downward filtration theory )
Recommended English as the medium of
1854
instruction for higher studies and vernaculars at
school level.

1857 Universities at Calcutta, Bombay and


Madras were set up.

ONLINE | DELHI | AHMEDABAD | BHOPAL | CHANDIGARH | GUWAHATI | HYDERABAD | JAIPUR | JODHPUR | LUCKNOW | PRAYAGRAJ | PUNE | RANCHI | SIKAR 33
STUDENTS NOTE

2017, 2018 and 2021 - UPSC repeated Lord Wellesley as a theme

2017
Which one of the following statements
2018 does not apply to the system of Subsidiary
Alliance introduced by Lord Wellesley?
With reference to educational institutions (a) To maintain a large standing army at
during colonial rule in India, consider the other's expense
following pairs:
(b) To keep India safe from Napoleonic
Institution Founder danger
1. Sanskrit College at Benaras - William (c) To secure a fixed income for the
Jones Company
2. Calcutta Madarsa - Warren (d) To establish British paramountcy over
Hastings the Indian States
3. Fort William College - Arthur
Wellesley 2021
Which of the pairs given above is/are Wellesley established the Fort William
correctly matched? College at Calcutta because
(a) 1 and 2 only (a) he was asked by the Board of Directors
(b) 2 only at London to do so
(c) 1 and 3 only (b) he wanted to revive interest in oriental
(d) 3 only learning in India
(c) he wanted to provide William Carey
and his associates with employment
(d) he wanted to train British civilians for
administrative purpose in India

ONLINE | DELHI | AHMEDABAD | BHOPAL | CHANDIGARH | GUWAHATI | HYDERABAD | JAIPUR | JODHPUR | LUCKNOW | PRAYAGRAJ | PUNE | RANCHI | SIKAR 34
Administrative Policies

YEAR 2017

Q10. Which one of the following statements does not apply to the system of
Subsidiary Alliance introduced by Lord Wellesley?
(a) To maintain a large standing army at other's expense
(b) To keep India safe from Napoleonic danger
(c) To secure a fixed income for the Company
(d) To establish British paramountcy over the Indian States

ANSWER C

SOURCE: BL Grover - Ch 10 - Lord Wellesley

EXPLANATION
The Company forced the states into a “subsidiary alliance”. According to the terms of this alliance,
Indian rulers were not allowed to have their independent armed forces. They were to be protected
by the Company, but had to pay for the “subsidiary forces” that the Company was supposed to
maintain for the purpose of this protection. If the Indian rulers failed to make the payment, then
part of their territory was taken away as penalty.
Napoleonic invasion of Egypt in the summer of 1798 offered Wellesly a useful tool to soften
London's resistance to expansion, although he never believed for a moment that there was any
danger of a French invasion of British India either over land from Egypt or a naval attack round the
Cape of Good Hope. However, to assuage London's concerns he evolved the policy of 'Subsidiary
Alliance', which would only establish control over the internal affairs of an Indian state, without
incurring any direct imperial liability.
There was no component of fixed income under this system.
Hence option (c) is the correct answer.

Administrative Policies

YEAR 2017

Q11. The objective of the Butler Committee of 1927 was to

(a) Define the jurisdiction of the Central and Provincial Governments.


(b) Define the powers of the Secretary of State for India.
(c) Impose censorship on national press.
(d) Improve the relationship between the Government of India and the Indian States.

ANSWER D
SOURCE: Spectrum A Brief History Of Modern India

EXPLANATION
The Report of the Butler Committee on the relations between the Indian States and British India.
The Committee advise that the Viceroy (instead of the Governor-General in Council) should
represent the Crown in all dealings with the States. Hence option (d) is the correct answer.
This proposed change is comparable with the new Imperial arrangement by which the
Governor-General of a Dominion is High Commissioner for the Dominion's Protectorates.
ONLINE | DELHI | AHMEDABAD | BHOPAL | CHANDIGARH | GUWAHATI | HYDERABAD | JAIPUR | JODHPUR | LUCKNOW | PRAYAGRAJ | PUNE | RANCHI | SIKAR 35
The Committee sympathize with the fear of the Native Princes that their States might pass without
their consent under a new Government in British India responsible to an Indian Legislature, and they
plainly express the opinion that no such transference should be made without the consent of the
Princes.

Administrative Policies

YEAR 2021

Q12. Wellesley established the Fort William College at Calcutta because


(a) he was asked by the Board of Directors at London to do so
(b) he wanted to revive interest in oriental learning in India
(c) he wanted to provide William Carey and his associates with employment
(d) he wanted to train British civilians for administrative purpose in India

ANSWER D

SOURCE: Old NCERT Chapter VI Administrative Organisation and Social and Cultural Policy

EXPLANATION

Fort William College was established on 18 August 1800 by Lord Richard Wellesley (d. 1837),
Governor General of Bengal, in order to provide instruction in the vernacular languages of India to
the civil and military officials of the East India Company. It was named after King William III of
England.
The civil servants received three year training in literature, science and languages in India, before
getting their civil posting.
Lord Wellesley wanted to train British civillian for administrative prupose in india. British officials
would be made familiar with the local languages, etc so that their administrative work would
become easier as it involved interaction with the Indian natives.
The college, however, could not continue for long due to fear among the members of the Court of
Directors that such a training programme might shift the loyalties of the civil servants from London
to Calcutta and ultimately resulted into its closure in 1802. Later an East India College was estab-
lished at Hertford near London in 1805, which was later moved to Haileybury in 1809, where a two
year training course was imparted to the civil servants. Hence option (d) is correct answer

ONLINE | DELHI | AHMEDABAD | BHOPAL | CHANDIGARH | GUWAHATI | HYDERABAD | JAIPUR | JODHPUR | LUCKNOW | PRAYAGRAJ | PUNE | RANCHI | SIKAR 36
A PRE-GLIMPSE
ON LORD WELLESLEY

The policy of Subsidiary Alliance was used by Lord Wellesley, who


arrived in India as Governor-General in 1798 and followed a forward
policy to bring as many states as possible under the British control.
To achieve this objective he relied on the methods of:

(a) Subsidiary Alliance.


(b) Outright War.
(c) Assumption of the Territories of Previously Subordinated Rulers.
Out of these methods, the system of Subsidiary Alliance played a very important part in the
expansion of the Company’s dominions and resulted into adding of many new territories to the
Company’s possessions.
Lord Wellesley, however, did not invent the system of Subsidiary Alliance as the system existed
long before him. The system of hiring European troops to Indian Princes at the latter’s expense
was started by Dupleix. The British adopted this system, where from Robert Clive to almost every
Governor-General had applied this system. The contribution of Lord Wellesley was that he
greatly developed it and applied it in almost every case of the Indian States.
Provisions Under Subsidiary Alliance: Following points were made under the system of
Subsidiary Alliance:
The Indian states were made to surrender their foreign relation to the Company and their
negotiations with other Indian states were also to be negotiated through the Company.

The states were expected to maintain a British army stationed within their own state. The state
was required to pay a tribute to the Company for the maintenance of this army or cede a part
of their territory for the upkeep of the army.
A British Resident was to be posted at the court of the Indian state.
The Indian states were debarred from employing any European in its service without the
consultation of the Company.
The Company agreed not to intervene in the internal affairs of the complying Indian states.
The Company promised to protect the Indian states from any sort of internal and external
threats.
In case of any conflict the Indian states had to agree to the decision of the Company.
The Indian states would have to acknowledge the Company as the “Paramount Power”.

Imposition of Subsidiary Alliance on Indian States:

The states that Surat,


accepted the The Rajput States
Subsidiary Alliance of Jodhpur, Jaipur,
were the Macheri, Bundi,
Nizam of Bharatpur (1818)
Hyderabad (1798 Holkar (last
and 1800), Maratha
Mysore (1799), confederacy to
Tanjore (1799), accept subsidiary
alliance in 1818).
Awadh (1801),
Peshwa (1801),
Bhonsle (1803),
Sindhia (1804),

ONLINE | DELHI | AHMEDABAD | BHOPAL | CHANDIGARH | GUWAHATI | HYDERABAD | JAIPUR | JODHPUR | LUCKNOW | PRAYAGRAJ | PUNE | RANCHI | SIKAR 37
SWADESHI MOVEMENT
Total No. of Questions: 3

Swadeshi Movement, Total No.of Questions: 3

1 2
Questions Question

Movement Based Current Affair


Based

Movement Based

Year 2019

Q1. With reference to Swadeshi Movement consider the following statements:


1. It contributed to the revival of the indigenous artisan crafts and industries.
2. The National Council of Education was established as a part of Swadeshi Movement.
Which of the statements given above is/are correct?
(a) 1 only
(b) 2 only
(c) Both 1 and 2
(d) Neither 1 nor 2

ANSWER C

SOURCE: Old NCERT, Class XII, Modern India.

EXPLANATION

The Swadeshi Movement had its genesis in the anti-partition movement which was started to
oppose the British decision to partition Bengal.
The Swadeshi movement sought to oppose British rule and encourage the ideas of self-help,
swadeshi enterprise, national education, and use of Indian languages. Hence statement 1 is
correct.
The Swadeshi Movement was the great emphasis given to self-reliance or ‘Atmasakti’ as a
necessary part of the struggle against the Government.
One of the major planks of the programme of self-reliance was Swadeshi or national education. So,
in August 1906, the National Council of Education was established. Its objective was to organize a
system of Education Literary; Scientific and Technical — on National lines and under National control
from the primary to the university level. Hence statement 2 is correct.
Hence option (c) is the correct answer.

ONLINE | DELHI | AHMEDABAD | BHOPAL | CHANDIGARH | GUWAHATI | HYDERABAD | JAIPUR | JODHPUR | LUCKNOW | PRAYAGRAJ | PUNE | RANCHI | SIKAR 38
Current Affair Based

Year 2022

Q2. With reference to the book "Desher Kather" written by Sakharam Ganesh
Deuskar during the freedom struggle, consider the following statement :

(a) 1 and 2 only


(b) 2 and 3 only
(c) 1 and 3 only
(d) 1, 2 and 3

ANSWER A

SOURCE: A History of Modern India-Ishita Banerjee-Dube.

STUDENTS NOTE
Motivation: 150th birth anniversary of Sakharam Ganesh
Deuskar in 2019.

EXPLANATION
Sakharam Ganesh Deuskar (1869-1912) was a close associate of Sri Aurobindo.
He published a book entitled Desher Katha describing in exhaustive detail the British commercial
and industrial exploitation of India.
This book had an immense repercussion in Bengal, captured the mind of young Bengal and assisted
more than anything else in the preparation of the Swadeshi movement.
Published first in June 1904, Desher Katha sold ten thousand copies in four editions within the year.
The fifth edition came out in 1905. The government of bengal banned the book in 1910 and confiscat-
ed all the copies.
Sakharam Ganesh Deuskar popularized the ideas of Naoroji and Ranade and promoted swadeshi in
a popular idiom.
His text, titled Desher Katha (Story of the Nation/Country), written in 1904, warned against the
colonial state’s ‘hypnotic conquest of the mind'. It inspired swadeshi street plays and folk songs,
and had become a mandatory text for an entire generation of swadeshi activists. Hence statement
1 and statement 2 is correct.
It was remarkable that in spite of this general growth of ‘national’ and regional awakening and
‘national’ consciousness, there was no word in Bengali for ‘nation’.
The term swadeshi—of one’s own desh—offered the best illustration of the use of desh, which orig-
inally meant place of origin; place in a geographic, social, linguistic and cultural sense, to refer to
the nation. Such use had featured in the writings of Rabindranath and other urban Bengalis from the
late-nineteenth century. It must also have been prevalent in Marathi.
Deuskar used desh to mean nation. The overlap of place of origin and nation, as well as jati (literally
birth, family or caste) and nationality inflected understandings of the nation/country in distinct ways.
Moreover, the constant overlap and conflation of Bengal and India, and Bengalis and Indians in the
use of desh and jati, added further twists to notions of nationalism. Hence statement 3 is not correct.

ONLINE | DELHI | AHMEDABAD | BHOPAL | CHANDIGARH | GUWAHATI | HYDERABAD | JAIPUR | JODHPUR | LUCKNOW | PRAYAGRAJ | PUNE | RANCHI | SIKAR 39
Current Affair Based

Year 2023

Q3. Consider the following statements :


Statement-I : 7th August is declared as the National Handloom Day.
Statement-II : It was in 1905 that the Swadeshi Movement was launched on the same day.
Which one of the following is correct in respect of the above statements?
(a) Both Statement-I and Statement-II are correct and Statement-II is the
correct explanation for Statement-I
(b) Both Statement-I and Statement-II are correct and Statement-II is not
the correct explanation for Statement-I
(c) Statement-I is correct but Statement-II is incorrect
(d) Statement-I is incorrect but Statement-II is correct

ANSWER A

SOURCE: https://indianexpress.com/article/lifestyle/art-and-culture/ national-


handloom -day- 2022-how-self-help-groups-sustain-indian-handicrafts-
handlooms-har-ghar-tiranga-8074511/

STUDENTS NOTE
Motivation: Har Ghar Tiranga Campaign and National Handloom Day was in
the news. The Ministry of Textiles, on its Twitter handle, posted a few select
pictures of Prime Minister Narendra Modi wearing the local traditional attire of
some Indian states.

EXPLANATION

On the occasion of National Handloom Day - observed every year on August 7 —The day
commemorates the Swadeshi movement that began in 1905 and was first observed in 2015,
marking its centenary celebration.
The formal proclamation of the Swadeshi Movement took place on 7th August 1905 in a meeting
held at the Calcutta Town Hall.

ONLINE | DELHI | AHMEDABAD | BHOPAL | CHANDIGARH | GUWAHATI | HYDERABAD | JAIPUR | JODHPUR | LUCKNOW | PRAYAGRAJ | PUNE | RANCHI | SIKAR 40
A PRE-GLIMPSE
TOWARDS SWADESHI MOVEMENT

The Partition of Bengal was announced in 17th July 1905 which set-off an Anti-Partition
Movement.

Anti partition movement and swadeshi movement started


On August 7, 1905, with the passage of the Boycott Resolution in a massive meeting held
in the Calcutta Townhall, the formal declaration of Swadeshi Movement was made.

on october 16 1905 the day the partition formally came into force, was observed as a day of
mourning throughout Bangal

Congress session related to swadeshi


movement

Banaras session 1905 Calcutta session 1906


In December 1905, at the Benaras session By 1905, the movement came to be
of the Indian National Congress presided spearheaded by the Extremists who
over by Gokhale, the Moderate- Extremist emboldened by the 1906 Calcutta
differences came to the fore. session's declaration of Swaraj gave a call
The Extremists wanted to extend the for passive resistance.
Boycott and Swadeshi Movement to Leaders like Aurobindo Ghosh, Bipin
regions outside Bengal and also to include Chandra Pal and Brahmabandhab
all forms of associations (such as Upadhyay made India's freedom
government service, law courts, legislative (swaraj) the central question in Indian
councils, etc.) politics.
The Extremists wanted a strong resolution The programme at this stage included
supporting their programme at the four things:
Benaras session. Boycott of British goods &
The Moderates, were not in favour of institutions (boycott of government
extending the movement beyond Bengal schools and colleges, government
and were totally opposed to boycott of services, courts, legislative councils,
councils and similar associations municipalities, government titles
As a compromise, a relatively mild and so on).
resolution condemning the partition of Development of their indigenous
Bengal and the reactionary policies of alternatives.
Curzon and supporting the swadeshi and Violation of unjust laws.
boycott programme in Bengal was
passed. This succeeded in averting a split Violent agitation if necessitated by
for the moment British repression.

ONLINE | DELHI | AHMEDABAD | BHOPAL | CHANDIGARH | GUWAHATI | HYDERABAD | JAIPUR | JODHPUR | LUCKNOW | PRAYAGRAJ | PUNE | RANCHI | SIKAR 41
SWADESHI MOVEMENT

The Swadeshi
movement sought to
oppose British rule To fight for swaraj, the Some also suggested
and encourage the radicals advocated that "revolutionary
ideas of self-help, mass mobilisation violence" would be
swadeshi enterprise, and boycott of British necessary to
national education, institutions and overthrow British rule.
and use of Indian goods.
languages.

Major leaders:
Finally, in 1911, the Lokamanya Tilak in
partition of Bengal Poona and Bombay, Ajit
was annulled in order Singh and Lala Lajpat
to curb the menace Rai in Punjab, Syed
of revolutionary Haidar Raza in Delhi
terrorism. and Chidambaram
Pillai in Madras
presidency.

Section's Participating Section's Not Participating

Students: of schools and colleges Vast majority of the working


Women: mainly belonging to urban middle class participated class
in picketing Peasantry:especially the
Educated middle class peasantry Muslim

Some members of the landed aristocracy Muslims: upper and middle


class (Nawab Salimullah of
Some Representative of commercial and mercantile interests Dacca and Muslim League
Some Muslims: like Abdul Rasul, Hasrat Mohani, Liaqat supported Partition of Bengal)
Hussain, Guznavi, Maulana Azad (joined revolutionary terrorist
group)

ONLINE | DELHI | AHMEDABAD | BHOPAL | CHANDIGARH | GUWAHATI | HYDERABAD | JAIPUR | JODHPUR | LUCKNOW | PRAYAGRAJ | PUNE | RANCHI | SIKAR 42
IMPACT OF SWADESHI MOVEMENT

New Forms of Struggle

Boycott of Foreign Goods: It included boycott and public burning of foreign cloth and
foreign-made salt or sugar.
Public Meetings and Processions: They emerged as major methods of mass mobilisation and
popular expression.
Corps of Volunteers or 'Samitis': These samitis generated political consciousness among the
masses through magic lantern lectures, swadeshi songs, providing physical and moral training
to their members.
Imaginative use of Traditional Popular Festivals and Melas: Tilak's Ganapati and Shivaji
festivals became a medium of swadeshi propaganda not only in western India, but also in
Bengal.
Emphasis given to Self-Reliance: This implied re-assertion of national dignity, honour and
confidence and social and economic regeneration of the villages.
Programme of Swadeshi or National Education: Bengal National College, inspired by Tagore's
Shantiniketan, was set up with Aurobindo Ghosh as its principal. Soon national schools and
colleges sprang up in various parts of the country.
Swadeshi or Indigenous Enterprises: The swadeshi spirit also found expression in the
establishment of swadeshi textile mills, soap and match factories, tanneries, banks, insurance
companies, shops, etc.
Impact in the Cultural Sphere: Tagore's Amar Sonar Bangla written on this occasion was later
inspired the liberation struggle of Bangladesh. In Tamil Nadu, Subramania Bharati wrote
Sudesha Geetham. In painting, Abanindranath Tagore broke the domination of Victorian
naturalism.

Annulment of Partition of Bengal


It was decided in 1911 (Delhi Durbar) under the viceroyship of Lord Hardinge II.
Lord Minto II was appointed the Viceroy of India in the year 1905 after the resignation of Lord
Curzon and served office till 1910. office till
It was decided to annul the partition of Bengal in 1911 mainly to curb the menace of
revolutionary terrorism.
The annulment came as a rude shock to the Muslim political elite.
It was decided to shift the capital to Delhi as a sop to the muslims as Delhi was associated
with muslim glory
Bihar and Orissa were taken out of Bengal and Assam was made a separate province.

ONLINE | DELHI | AHMEDABAD | BHOPAL | CHANDIGARH | GUWAHATI | HYDERABAD | JAIPUR | JODHPUR | LUCKNOW | PRAYAGRAJ | PUNE | RANCHI | SIKAR 43
Gandhian phase

Gandhian Phase, Total No.of Questions: 6

1 1 2 1 1
Questions Question Questions Questions Question

Satyagraha Important Gandhi Current Affairs Movement


Association\ British Linkage Based
Organizations Negotiation

Satyagraha

Year 2018

Q1. Which one of the following is a very significant aspect of the Champaran
Satyagraha?
(a) Active all-India participation of lawyers, students and women in the
National Movement
(b) Active involvement of Dalit and Tribal communities of India in the
National Movement
(c) Joining of peasant unrest to India's National Movement
(d) Drastic decrease in the cultivation of plantation crops and
commercial crops

ANSWER C

SOURCE: Old NCERT Bipin Chandra

STUDENTS NOTE
Champaran Satyagraha Completed 100 years in 2017

EXPLANATION

Champaran Satyagraha of 1917 opened a new phase in the national movement by joining it to
the great struggle of the Indian peasantry for bread and land.
It was the first peasant movement to have garnered nation wide attention. The Champaran
Satyagraha yoked the peasant unrest to the freedom struggle. Hence option (c) is the correct
answer.

ONLINE | DELHI | AHMEDABAD | BHOPAL | CHANDIGARH | GUWAHATI | HYDERABAD | JAIPUR | JODHPUR | LUCKNOW | PRAYAGRAJ | PUNE | RANCHI | SIKAR 44
Important Association\Organizations

Year 2018

Q2. In 1920, which of the following changed its name to ""Swarajya Sabha""?
(a) All India Home Rule League
(b) Hindu Mahasabha
(c) South Indian Liberal Federation
(d) The Servants of India Society

ANSWER A

SOURCE: Uncoventional-Satyagraha by Savita Singh

EXPLANATION

Gandhi had become the president of the All India Home Rule League. Its name was changed to
'Swaraj Sabha'. A fourfold plan of constructive work was to be carried out through the Sabha.
Hindu-Muslim unity, spread of swadeshi including the use of charkha, spread of Hindustani as the
lingua franca of India, and the formation of linguistic provinces were the items he proposed to the
Sabha. Hence option (A) is correct answer.

Gandhi British Negotiation

Year 2019

Q3. With reference to the British colonial rule in India, consider the following statements:
1. Mahatma Gandhi was instrumental in the abolition of the system of ‘indentured labour’.
2. In Lord Chelmsford’s War Conference’, Mahatma Gandhi did not support the resolution
on recruiting Indians for World War.
3. Consequent upon the breaking of Salt Law by Indian people, the Indian National Congress
was declared illegal by the colonial rulers.
Which of the statements given above are correct?
(a) 1 and 2 only
(b 1 and 3 only
(c) 2 and 3 only
(d) 1, 2 and 3

ANSWER B

SOURCE: Spectrum A Brief History Of Modern India

EXPLANATION
Mahatma Gandhi was instrumental in the abolition of the system of ‘indentured labour’ in South
Africa.
In Lord Chelmsford’s War Conference’, Mahatma Gandhi did not support the resolution on recruiting
Indians for World War. Moderates supported the First World War.
Congress was declared illegal by British government after the launch of second phase of Civil
Disobedience Movement.

ONLINE | DELHI | AHMEDABAD | BHOPAL | CHANDIGARH | GUWAHATI | HYDERABAD | JAIPUR | JODHPUR | LUCKNOW | PRAYAGRAJ | PUNE | RANCHI | SIKAR 45
Additional Information: LORD CHELMSFORD WAR CONFERENCE
The War Conference was held to seek Indian participation in the war efforts. The conference
was convened in 1917 when the war had entered a critical phase. Germany had inflicted
crushing defeats on both the British and French troops in France. Russia's war effort had broken
down and the Russian Revolution was threatening its Czarist Government. At that moment,
Indian support became crucial for the British Empire. Overall around 1.3 million Indian troops
had served in World War 1.
The Conferences existed concurrently with the Imperial War Cabinet, which was a British body
for wartime coordination. It had representatives from all British colonies like Canada, India etc. It
recognised the contributions of colonies to war efforts.
It was convened in Delhi under Lord Chelmsford who was the Viceroy then.
It was held to persuade national leaders to help the government in war purposes.
It mainly included recruiting soldiers for participating in the war.
Gandhiji attended the conference and accepted to give his services. He later started an active
recruiting campaign in Kaira (Kheda) District. Gandhiji believed that only the absolutely
unconditional and whole-hearted co-operation with the government on the part of educated
India will bring Swaraj.

Gandhi British Negotiation

Year 2020
Q4. The Gandhi-Irwin Pact included which of the following?
1 Invitation to Congress to participate in the Round Table Conference
2. Withdrawal of Ordinances promulgated in connection with the Civil Disobedience Movement
3. Acceptance of Gandhiji's suggestion for enquiry into police excesses.
4. Release of only those prisoners who were not charged with violence

Select the correct answer using the code given below:


(a) 1 only
NOTE
UPSC DROPPED THIS
(b) 1, 2 and 4 only QUESTION - NO OFFICIAL
(c) 3 only ANSWER KEY WAS RELEASED.
THE TOPIC HOWEVER, IS
(d) 2, 3 and 4 only IMPORTANT FROM
PRELIMS POINT OF
VIEW

SOURCE: NCERT Themes in Indian History part-III Class 12th Chapter-Mahatma Gandhi
and the Nationalist Movement Civil Disobedience and Beyond.

EXPLANATION
5 March 1931, the Gandhi-Irwin Pact was signed by Gandhiji on behalf of the Congress and by Lord
Irwin on behalf of the Government.
The pact placed the Congress on an equal footing with the Government. 2
The terms of the agreement included immediate release of all political prisoners not convicted of
violence; remission of all fines not yet collected; return of all lands not yet sold to third parties;
lenient treatment to those government servants who had resigned; right to make salt in coastal
ONLINE | DELHI | AHMEDABAD | BHOPAL | CHANDIGARH | GUWAHATI | HYDERABAD | JAIPUR | JODHPUR | LUCKNOW | PRAYAGRAJ | PUNE | RANCHI | SIKAR 46
village for personal consumption (not for sale); right to peaceful and non- aggressive picketing;
withdrawal of emergency ordinances.
The viceroy, however, turned down two of Gandhi's demands:
(i) public inquiry into police excesses, and
(ii) commutation of Bhagat Singh and his comrades' death sentence to life sentence.

Gandhi on behalf of the Congress agreed:


(i) to suspend disobedience the civil movement.
(ii) to participate in the next Round Table Conference.

Current Affairs Linkage

Year 2021

Q5. Who among the following is associated with 'Song from Prison', a translation of
ancient Indian religious lyrics in English?
(a) Bal Gangadhar Tilak
(b) Jawaharlal Nehru
(c) Mohandas Karamchand Gandhi
(d) Sarojini Naidu

ANSWER C

SOURCE:
Unconventional
- https://www.raptisrarebook s.com/product /songs-from- prison- translations-
of-indian- lyrics-made- in-jail- mohandas-k- gandhi-first- edition-rare

EXPLANATION
‘Songs from Prison' is a book of translation from Sanskrit hymns and lyrics, from the Upanishads
and other scriptures.
Mahatma Gandhi made these translations during his incarceration in Yeravada Prison in 1930.
Translations of works by Manu, Tulsidas, Kabir, Nanak, Mirabai, Ramdas, Tukaram and many other
poets were used mainly for his English friends and especially for Mirabehn.

STUDENTS NOTE
In Jan. 2021 Maharashtra Govt. Started Jail Tourism Initia-
tive from Yerawada Jail (Pune). Gandhiji while he was in
Yerawada jail wrote the book ‘Songs in Prison’.

ONLINE | DELHI | AHMEDABAD | BHOPAL | CHANDIGARH | GUWAHATI | HYDERABAD | JAIPUR | JODHPUR | LUCKNOW | PRAYAGRAJ | PUNE | RANCHI | SIKAR 47
Movement Based

YEAR 2021

Q6. With reference to 8th August, 1942 in Indian history, which one of the following
statements is correct?

(a) The Quit India Resolution was adopted by the AICC.


(b) The Viceroy's Executive Council was expanded to include more Indians.
(c) The Congress ministries resigned in seven provinces.
(d) Cripps proposed an Indian Union with full Dominion Status once the Second World War
was over.

ANSWER A

SOURCE: Old NCERT Bipin Chandra

EXPLANATION

In July 1942, the Congress Working Committee met at Wardha and resolved that it would authorise
Gandhi to take charge of the non-violent mass movement. The resolution generally referred to as
the ‘Quit India’ resolution. Proposed by Jawaharlal Nehru and seconded by Sardar Patel, it was to
be approved by the All India Congress Committee meeting in Bombay in August. The Quit India
Resolution was ratified at the Congress meeting at Gowalia Tank, Bombay, on August 8, 1942.
Hence option (a) is the correct answer.

QUIT INDIA MOVEMENT

Immediate causes
Failure of Cripps Mission.
Advancement of Japanese forces to Indian shore shattering the myth of
British invincibility. Public discontent against Wartime hardships.

Quit India Resolution


The Quit India Resolution was ratified at the Congress meeting at
Gowalia Tank, Bombay, on August 8, 1942.
Gandhi’s General Instructions to Different Sections :
(a) Government servants: Do not resign but declare your allegiance
to the Congress.
(b) Soldiers: Do not leave the Army but do not fire on compatriots.
(c) Peasants: If zamindars are anti-government, pay mutually agreed
rent, and if zamindars are pro-Govt then do not pay rent.

ONLINE | DELHI | AHMEDABAD | BHOPAL | CHANDIGARH | GUWAHATI | HYDERABAD | JAIPUR | JODHPUR | LUCKNOW | PRAYAGRAJ | PUNE | RANCHI | SIKAR 48
Spread of the Movement
On 9th August, all top leaders of congress were arrested. Aruna Asaf Ali,
presided over the Congress committee session, and hoisted the flag.

Underground Activity: Many nationalists went underground and took to


subversive activities. Eg. Rammanohar Lohia, Jayaprakash Narayan,
Aruna Asaf Ali, Usha Mehta, Biju Patnaik, Chhotubhai Puranik, Achyut
Patwardhan, Sucheta Kripalani and R.P. Goenka. Usha Mehta started an
underground radio in Bombay.

Parallel Government was formed: eg. Ballia under Chittu Pandey ; Tamluk
(Midnapore) under Jatiya Sarkar (organised Vidyut Vahinis); Satara
(“Prati Sarkar”), under leaders like Y.B. Chavan, Nana Patil, etc.

Note: All other parties eg. ML, Hindu MahaSabha, Princely states, Communist (sympathetic to
Russian cause against NAZI attack) etc. stayed aloof to the movement. Further, to
condemn violence of State, Gandhi ji under took fast.

ADMISSION OPEN

ONLINE | DELHI | AHMEDABAD | BHOPAL | CHANDIGARH | GUWAHATI | HYDERABAD | JAIPUR | JODHPUR | LUCKNOW | PRAYAGRAJ | PUNE | RANCHI | SIKAR 49
A PRE-GLIMPSE
TO GANDHI JI AND HIS
CONTRIBUTION

MAHATMA GANDHI
BRIEF CHRONOLOGY

2 October 1869 1883 1888 1893


Birth at Porbandar, Marriage with Reached He landed at Durban (South
Gujarat. Kasturbai, aged 13. England. Africa) to sort out the legal
problems of his client Dada
Abdullah.

1903 1899 1894 1893


The newspaper Established Founded ∙ Gandhi himself faced
'Indian Opinion' Indian the 'Natal racial discrimination &
started by Ambulance Indian humiliation in South Africa.
Gandhiji in Corp during Congress'. ∙ He was thrown off
South Africa. the Boer War the first class carriage of the
train at Pietermartizburg Station.

1904 1906 1908 1909


Established 'Phoenix Took the The term 'Satyagraha' Wrote 'Hind
Settlement' in Natal. vow of adopted based on Swarajya' aboard the
It was a precursor Brahmacharya. Maganlal Gandhi's ship 'Kildonan
to the Tolstoy Farm. fomulation 'Sadagraha' Castle'

9 January 1915 1915 1912 1910


Gandhi returned Awarded the Gopal krishna Established
to India from 'Kaiser i Hind' Gokhale's South Tolstoy Farm.
South Africa. medal. Africa tour commenced.
Gopala Krishna Gokhale
was the political
mentor of Gandhi.

ONLINE | DELHI | AHMEDABAD | BHOPAL | CHANDIGARH | GUWAHATI | HYDERABAD | JAIPUR | JODHPUR | LUCKNOW | PRAYAGRAJ | PUNE | RANCHI | SIKAR 50
1917 1917 1918 1918
Sabarmati Champaran Ahmedabad Mill Kheda Satyagraha
Ashram Satyagraha (1917)— Strike (1918) — First (1918)—First
established. First Civil Hunger Strike. Non-Cooperation.
Disobedience.

1920 1920 1919 1919


Commencement Hunter Commission Massacre at Satyagraha against
of Non-Cooperation of 1920 appointed Jallianwala Bagh the Rowlatt Act-
Movement. to investigate the or Massacre of ∙ Gandhiji called for a
Jallianwala Bagh Amritsar. countrywide campaign
or Massacre of against the “Rowlatt Act”.
Amritsar. ∙ Civil disobedience against
specific laws, and courting
arrest & imprisonment.

1922 1922 1924 1925


Bardoli Taluka Policemen at Chauri- Presided over 'All India Spinners'
resolved against Chaura killed. Due to this the Belgaum Association'
payment of land incident Mahatma Congress. established by
revenue and Gandhi called off the Gandhiji.
Civil-Disobedience. Non-Cooperation
Movement.

1931 1930 1929 1925


Gandhi-Irwin Dandi March Purna Swaraj The publication of
pact signed. commences Resolution Autobiography -
from Satyagraha adopted The Story of My
Ashram. at Lahore Experiments with Truth'.
Congress. ('Satyana Prayogo
athava Atmakatha')

ONLINE | DELHI | AHMEDABAD | BHOPAL | CHANDIGARH | GUWAHATI | HYDERABAD | JAIPUR | JODHPUR | LUCKNOW | PRAYAGRAJ | PUNE | RANCHI | SIKAR 51
1931 1932 1932 1932
In London as the Premier Ramsay Poona Pact signed. 'Harijan Sevak
sole representative Macdonald (It gave depressed Sangh' established.
of the Congress at announced classes reserved
the Second Round the Communal seats in the
Table Conference. Award. It was to provincial and
grant separate central legislative
electorates for councils but to
different communities be voted in by the
including the general electorate)
depressed classes.
It is also known as
'McDonald Award'.

1946 1942 1936 1933


Three member 'Quit India' Travancore Commenced
British resolution temples publication
delegation reaches adopted; gave opened to of 'Harijan', 'Harijan
Delhi. (Stafford the call 'Do or Harijans. Sevak' (Hindi), and
Cripps, Die'. 'Harijanbandhu'
Pethick-Lawrence (Gujarati).
and A.V. Alexander)

1946 30 January 1948 1948


The Congress Working Godse Cremated on the banks
Committee adopted assassinates of the Yamuna by
resolution to accept Gandhi. Ramdas. Raj Ghat is
the formation of the a memorial dedicated to
Constituent Assembly. Gandhi in Delhi.

ONLINE | DELHI | AHMEDABAD | BHOPAL | CHANDIGARH | GUWAHATI | HYDERABAD | JAIPUR | JODHPUR | LUCKNOW | PRAYAGRAJ | PUNE | RANCHI | SIKAR 52
Gandhi ran four publications

Indian Young Navajivan Harijan


Opinion India

Gandhi’s early Rowlatt


movements in India Satyagraha

Localised struggles & Rowlatt satyagraha made


9th Jan-1915 identification with the Gandhiji a truly national leader
Gandhi's return masses.
to India
This day is celebrated as
Pravasi Bharatiya Diwas Rowlatt Act also known as
Champaran Anarchical & Revolutionary
in India since 2003. Satyagraha (1917):
crimes Act, 1919
PBD 2021 Theme :- First Civil Disobedience;
Contributing to Rajkumar Shukla
Athmanirbhar Bharat. requested Gandhiji to look
into the issues faced by It allowed detention of
Indigo planters. political prisoners
without trial for two years.
Tinkathia system whereby This act was met by wide
European planters forced spread anger and discontent
Indian peasants to grow among Indians, especially in
Indigo on 3/20 of the the Punjab region.
total land.

ONLINE | DELHI | AHMEDABAD | BHOPAL | CHANDIGARH | GUWAHATI | HYDERABAD | JAIPUR | JODHPUR | LUCKNOW | PRAYAGRAJ | PUNE | RANCHI | SIKAR 53
Ahmedabad Mill Strike
(1918): Massacre at Jallianwala
Bagh or Massacre of
First Hunger Strike.
Amritsar-1919
Dispute between the General Dyer opened fire on
mill owners & workers unarmed crowd killing
over the issue of thousands;
discontinuation of the People had gathered on
Plague bonus. Baisakhi day to protest
against the arrest of their
leaders (Saifuddin
Kitchlew & Satyapal)

Kheda Satyagraha
(1918):
First Non-Cooperation.
Rabindranath Tagore
The authorities refused
renounced his knighthood
to grant remission due
in protest.
to crop failure.
Crops failed due to
droughts in Kheda
(Gujarat).
Gandhi withdrew
the movement(satyagraha
against Rowlett) after this
incident. He called it
Himalayan Blunder.
He also returned the
Kaiser-i-Hind Gold medal.

Hunter commission of 1920


appointed to investigate
Jallianwala Bagh incident.

ONLINE | DELHI | AHMEDABAD | BHOPAL | CHANDIGARH | GUWAHATI | HYDERABAD | JAIPUR | JODHPUR | LUCKNOW | PRAYAGRAJ | PUNE | RANCHI | SIKAR 54
TRANSFER OF POWER
Total No. of Questions: 1

Cripps Mission

Year 2022

Q1. With reference to the proposals of Cripps Mission, consider the following statements:
1. The Constituent Assembly would have members nominated by the Provincial Assemblies as
well as the Princely States.
2. Any Province, which is not prepared to accept the new Constitution would have the right to sign
a separate aggrement with Britain regarding its future status.
Which of the statements given above is/are correct?

(a) 1 only
(b) 2 only
(c) Both 1 and 2
(d) Neither 1 nor 2

ANSWER B

SOURCE: Spectrum A Brief History Of Modern India

EXPLANATION

The Cripps mission was an attempt in late March 1942 by the British government to secure full
Indian cooperation and support for their efforts in World War II. The mission was headed by Sir
Stafford Cripps, a senior left-wing politician and government minister in the War Cabinet, who had
actively supported the Indian national movement.
The main proposals of the mission were as follows:
An Indian Union with a dominion status would be set up; it would be free to decide its relations with
the Commonwealth.
After the end of the war, a Constituent Assembly would be convened to frame a new constitution.
Members of this assembly would be partly elected by the provincial assemblies through
proportional representation and partly nominated by the princes.
Hence statement 1 is not correct.
The British government would accept the new constitution subject to two conditions: any
province not willing to join the Union could have a separate constitution and form a separate
Union, and the new constitution- making body and the British government would negotiate a
treaty to effect the transfer of power and to safeguard racial and religious minorities. Hence
statement 2 is correct.
In the meantime, defence of India would remain in British hands and the governor-general’s
powers would remain intact.

ONLINE | DELHI | AHMEDABAD | BHOPAL | CHANDIGARH | GUWAHATI | HYDERABAD | JAIPUR | JODHPUR | LUCKNOW | PRAYAGRAJ | PUNE | RANCHI | SIKAR 55
TIMELINE: INDIAN FREEDOM MOVEMENT FROM 1939 TO 1947

CWC at wardha: August offer promising Cripps Mission:


Congress did not support Dominion Status Promise of Dominion Status;
British during WW2 Constituent Assembly (India)

March,
1940 Oct, 1940

1939 August, March,


1940 1942

Pakistan Individual Satyagraha


Resolution was passed movement was launched
by Muslim League to affirm the rights to speech.

Launch of QIM Desai Liaqat pact, Three Upsurges:


Demanding immediate in pursuance to unity (Over INA trials against
British withdrawal between INC and ML, sentencing of Rashid Ali,
from India came but it was failed too. and RIN (Navy) Revolt).

1944 June, 1945

Aug,1942 1945 Winter of


1945-1946

Rajaji Formula to Wavell Plan:


solve deadlock between Formation of Indian
ML and INC. executive Council
which act as interim
government; negotiations
on new constitution.

ONLINE | DELHI | AHMEDABAD | BHOPAL | CHANDIGARH | GUWAHATI | HYDERABAD | JAIPUR | JODHPUR | LUCKNOW | PRAYAGRAJ | PUNE | RANCHI | SIKAR 56
Cabinet Mission Plan:
Mountbatten to draw constitution of India and
plan facilitate interim government.

Dec, 1945-
July, 1947 Feb, 1947 Jan, 1946

June, 1947 1946

Indian Attlee’s General election:


Independence Act statement INC emerged as
the largest party.

ADMISSION OPEN

ONLINE | DELHI | AHMEDABAD | BHOPAL | CHANDIGARH | GUWAHATI | HYDERABAD | JAIPUR | JODHPUR | LUCKNOW | PRAYAGRAJ | PUNE | RANCHI | SIKAR 57
Peasant Movements and Tribal Rebellions
Total No. of Questions: 03

Peasant Movements and Tribal Rebellions, Total No. of Questions: 3

2 1
Questions Question

Movement Based Current Affair Linkage

Current Affair Linkage

Year 2018

Q1. Who among the following were the founders of the "Hind Mazdoor Sabha"
established in 1948?
(a) B. Krishna Pillai, E.M.S. Namboodiripad and K.C. George
(b) Jayaprakash Narayan, Deen Dayal Upadhyay and M.N. Roy
(c) C.P. Ramaswamy Iyer, K. Kamaraj and Veeresalingam Pantulu
(d) Ashok Mehta, T.S. Ramanujan and G.G. Mehta

ANSWER D

STUDENTS NOTE Motivation to Ask this Question

In 2017, several Labour unions like those affiliated to the Communist


Party of India (Marxist), Shiv Sena and Congress such as the Trade Union
Joint Action Committee, the Centre of Indian Trade Unions affiliated to
CPM, the Bharatiya Kamgar Sena, affiliated to the Shiv Sena, the Indian
National Trade Union Congress affiliated to the Congress, the Hind
Mazdoor Sabha and other labour organisations addressed against
proposed amendments to Industrial Disputes Act.
Labour unions up in arms against proposed amendments to Industrial
Disputes Act
A state-level meeting of all labour unions would be held in Pune in the
next three weeks to decide the further course of action, including strike
and protests, said Utagi, adding they also opposed the amendments
made in the Factories Act, the Contract Labourers Act and the Shops
and Establishment Act.

SOURCE: http://www.hindmazdoorsabha.com/about-hms.php

ONLINE | DELHI | AHMEDABAD | BHOPAL | CHANDIGARH | GUWAHATI | HYDERABAD | JAIPUR | JODHPUR | LUCKNOW | PRAYAGRAJ | PUNE | RANCHI | SIKAR 58
EXPLANATION

The Hind Mazdoor Sabha (HMS) is a National Trade Union Centre in India. It was founded in Howrah
on 24.12.1948 by Socialist, Forward Bloc follower and independent unionists.
Its founders included Basawan Singh (Sinha), Ashok Mehta, R.S. Ruikar, Mani Benkara, Shibnath
Benerajee, R.K. Khedgikar, T.S. Ramanujam, VS. Mathur, G.G. Mehta. Mr. R.S. Ruikar was elected
president and Ashok Mehta as its General Secretary. Hence option (d) is the correct answer.
The independence of trade unions from the Government, the employers and political parties is the
basic feature of HMS.
The HMS absorbed the Royists Indian Federation of Labour and the Socialist Hind Mazdoor
Panchayat.
The Hind Mazdoor Sabha was intended to be a third force in Indian trade unionism, balancing
between INTUC on one side and AITUC on the other.
INTUC was discarded because it was believed to be under the control of the government and
AITUC was rejected because it was controlled by the Communist Party.

Movement Based

Year 2018

Q2. After the Santhal Uprising subsided, what was/were the measure/measures
taken by the colonial government?
1. The territories called 'Santhal Paraganas' were created.
2. It became illegal for a Santhal to transfer land to a non-Santhal.

Select the correct answer using the code given below:


(a) 1 only
(b) 2 only
(c) Both 1 and 2
(d) Neither 1 nor 2

ANSWER C

SOURCE: Shekhar Bandopadhyaya's "Plassey To Partition" -Chapter - Reform &


Rebellion

EXPLANATION

Statement 1 is correct: It was after the Santhal Revolt (1855-56) that the Santhal Pargana was
created, carving out 5,500 square miles from the districts of Bhagalpur and Birbhum. The colonial
state hoped that by creating a new territory for the Santhals and imposing some special laws within
it, the Santhals could be conciliated.
Statement 2 is correct: To protect economic basis of Santhal society,special regulations governed
transfer of land. It became illegal for a Santhal to transfer land to a non-Santhal.

ONLINE | DELHI | AHMEDABAD | BHOPAL | CHANDIGARH | GUWAHATI | HYDERABAD | JAIPUR | JODHPUR | LUCKNOW | PRAYAGRAJ | PUNE | RANCHI | SIKAR 59
Movement Based

Year 2020

Q3. With reference to the history of India, "Ulgulan" or the Great Tumult is the
description of which of the following events?
(a) The Revolt of 1867
(b) The Mappila Rebellion of 1921
(c) The Indigo Revolt of 1859 - 60
(d) Birsa Munda's Revolt of 1899-1900

ANSWER D
SOURCE: Civil Rebellions and Tribal Uprising,
Bipin Chandra - India's struggle for independence, Chapter 2

EXPLANATION

Munda Rebellion is one of the prominent 19th century tribal rebellions in the subcontinent. Birsa
Munda led this movement in the region south of Ranchi in 1899-1900.
The ulgulan, meaning 'Great Tumult', sought to establish Munda Raj and independence. The
Mundas traditionally enjoyed a preferential rent rate as the khuntkattidar or the original clearer of
the forest. But in course of the 19th century they had seen this khuntkatti land system being eroded
by the jagirdars and thikadars coming as merchants and moneylenders. Hence option (d) is the
correct answer.
The government attempted to redress the grievances of the Mundas through the survey and
settlement operations of 1902-10.
The Chhotanagpur Tenancy Act of 1908 provided some recognition to their khuntkatti rights and
banned beth begari. Chhotanagpur tribals won a degree of legal protection for their land rights.

ADMISSION OPEN

ONLINE | DELHI | AHMEDABAD | BHOPAL | CHANDIGARH | GUWAHATI | HYDERABAD | JAIPUR | JODHPUR | LUCKNOW | PRAYAGRAJ | PUNE | RANCHI | SIKAR 60
A PRE-GLIMPSE
TRIBAL AND CIVIL UPRISINGS

A. BENGAL AND NORTH EAST

Name
Sanyasi Revolt (late 18th Century)

Chuar Uprising (also called Revolt of the


Jungle Mahal)

Kol Uprising (1831)

Ho and Munda Uprisings (1820-1837)

Santhal Rebellion (1855-56)

Khond Uprising (1837-1856)

Ahom Revolt (1828)

Khasi Uprising (1833)

Pagal Panthis (late 18th Century)

Farazi Revolt (1838-1857)

Munda Revolt (1899-1900)

B. WESTERN INDIA

Name

Bhil Uprising (1817-1846)

Cutch Rebellion (1816)

Waghera Rising (1818-1820)

Koli Rising (1829)

Ramosi Risings (1822)

Surat Salt Agitations (1844)

Kolhapur and Savantvadi Revolts (1838)

ONLINE | DELHI | AHMEDABAD | BHOPAL | CHANDIGARH | GUWAHATI | HYDERABAD | JAIPUR | JODHPUR | LUCKNOW | PRAYAGRAJ | PUNE | RANCHI | SIKAR 61
C. SOUTH INDIA

Name
Revolt of Raja of Vizianagaram (1794)

Poligars Revolt (1799))

Diwan Velu Tampi's Revolt (1805)

Rampa Revolt (1879)

D. NORTH INDIA

Name
Wahabi Movement (19th Century)
Kuka Revolt (1840)

ONLINE | DELHI | AHMEDABAD | BHOPAL | CHANDIGARH | GUWAHATI | HYDERABAD | JAIPUR | JODHPUR | LUCKNOW | PRAYAGRAJ | PUNE | RANCHI | SIKAR 62
SOCIO-RELIGIOUS MOVEMENTS
Total No. of Questions: 3

Personality Based Question

Year 2020

Q1. In the context of Indian history, the Rakhmabai case of 1884 revolved around
1. women's right to gain education
2. age of consent
3. restitution of conjugal rights

Select the correct answer using the code given below:


(a) 1 and 2 only
(b) 2 and 3 only
(c) 1 and 3 only
(d) 1, 2 and 3

ANSWER B

SOURCE: PLASSEY TO PARTITION by Shekhar Bandopadhyay: Chapter - Early Nationalism

STUDENTS NOTE
In 2017 Google Doodle Was Dedicated To Rakhmabai Bhikaji

ONLINE | DELHI | AHMEDABAD | BHOPAL | CHANDIGARH | GUWAHATI | HYDERABAD | JAIPUR | JODHPUR | LUCKNOW | PRAYAGRAJ | PUNE | RANCHI | SIKAR 63
EXPLANATION

Dr. Rakhmabai Bhikaji was a pioneer in the field of medicine and women’s rights in the 19th
century.
Her efforts to be granted the right to choose was instrumental in raising the age of consent for
women in 1891. She went on to study in the London school of Medicine for Women in 1889. When she
came back to India to work in a hospital in 1894, she became India’s first practicing lady doctor.

Rakhmabai got married at 11-years of age to the 19-year old Dadaji Bhikaji. As was convention at
the time, she stayed at her parents house, this was the time she spent in educating herself under
the guidance of her stepfather.

When Rakhmabai was still in school, her husband, Dadaji, insisted that Rakhmabai come and live
with him in his house. Rakhmabai, not one to blindly follow convention, refused.

Dadaji soon filed a petition in the court of law. Early in 1884, one of India’s most influential and
publicized trials began.

Rukhmabai asked ‘freedom’ from her non consensual marriage, seeking a legal divorce. Her
stand gave rise to a nation-wide debate over infant and non-consensual marriage.

The legal and social controversies provoked by the case revolved around notions of colonial law,
marriage and conjugality, and the prospect of state intervention. For the first time, the social
orthodoxy, which had so far resisted state intervention in ‘family’ and ‘religion’, appealed to colonial
law to discipline the disobedient wife.

Behramji Malabari and Pandita Ramabai came to her defense and formed the Rakhmabai
Defense Committee. The case spanned 4 years until Dadaji was “compensated” in 1888, outside of
court. Hence option (b) is the correct answer.

Personality Based Question

Year 2020

Q.2 The Vital-Vidhvansak, the first monthly journal to have the untouchable people as its target
audience was published by

(a) Gopal Baba Walangkar


(b) Jyotiba Phule
(c) Mohandas Karamchand Gandhi
(d) Bhimrao Ramji Ambedkar

ANSWER A

SOURCE: PLASSEY To PARTITION by Shekhar Bandopadhyay: Chapter-Early Nationalism

EXPLANATION
Gopal Baba Walangkar, also known as Gopal Krishna, (ca. 1840-1900) was a contemporary of
Mahatma Phule & was instrumental in anti-caste movement in Maharashtra.
He developed a racial theory to explain the oppression and also published the first journal targeted
at the untouchable people.
Walangkar extended Phule's version of this racial theory, that the untouchable people of India

ONLINE | DELHI | AHMEDABAD | BHOPAL | CHANDIGARH | GUWAHATI | HYDERABAD | JAIPUR | JODHPUR | LUCKNOW | PRAYAGRAJ | PUNE | RANCHI | SIKAR 64
were the indigenous inhabitants and that the Brahmin people were descended from Aryans who
had invaded the country.
In 1888, Walangkar began publishing the monthly journal titled Vital-Vidhvansak (Destroyer of
Brahmanical or Ceremonial Pollution), which was the first to have the untouchable people as its
target audience.

Personality Based Question

Year 2021

Q.3 Who among the following was associated as Secretary with Hindu Female School which later
came to be known as Bethune Female School?

(a) Annie Besant


(b) Debendranath Tagore
(c) Ishwar Chandra Vidyasagar
(d) Sarojini Naidu

ANSWER C

SOURCE: Bipin chandra Modern India (Old NCERT)

EXPLANATION
With encouragement from and participation of like-minded social reformers like Ramgopal Ghosh,
Raja Dakshinaranjan Mukherjee and Pandit Madan Mohan Tarkalankar, Bethune established
Kolkata's first school for girls in 1849 called the Hindu Female School.
Bethune passed away in 1851. In 1856, the Government took charge of the Hindu Female School, later
renamed as Bethune School.
The Managing Committee of the school was then formed and Pandit Ishwarchandra Vidyasagar,
the celebrated social reformer responsible for the eradication of the custom of Sati and a relentless
supporter of women's emancipation was made the Secretary. Hence option (c) is the correct
answer.

ONLINE | DELHI | AHMEDABAD | BHOPAL | CHANDIGARH | GUWAHATI | HYDERABAD | JAIPUR | JODHPUR | LUCKNOW | PRAYAGRAJ | PUNE | RANCHI | SIKAR 65
PRE-GLIMPSE ON SOCIO RELIGIOUS
MOVEMENTS

GAVE RISE TO
INDIAN THE PROCESS OF DESIRE FOR
RENAISSANCE REAWAKENING. REFORMS.

REFORM MOVEMENTS (AMONG HINDUS)

PAN INDIA MOVEMENT


Ramkrishna Movement
Arya Samaj
Theosophical Movement

1. Student’s Library and


1. Brahmo Samaj.
Scientific Society.
2. Tattvabodhini Sabha.
2. Paramhansa Mandalis
3. Brahmo Samaj of India
3. Satyashodhak Samaj
4. Young bengal Movement.
4. Servants of India Society
5. Prarthana Samaj.

1. SNDP Movement.
2. Vokkaliga Sangha.
3. Justice Movement.
4. Self-respect Movement.
5. Temple Entry Movement.

ONLINE | DELHI | AHMEDABAD | BHOPAL | CHANDIGARH | GUWAHATI | HYDERABAD | JAIPUR | JODHPUR | LUCKNOW | PRAYAGRAJ | PUNE | RANCHI | SIKAR 66
OTHER PROMINENT PERSONALITIES (REGION-WISE):

EASTERN INDIA:
AKSHAY KUMAR DUTTA:

in
s.
ia
Progressive Bengali thinker

on
behind the reformist

si
vi
Brahmo Samaj.

d@
Formulated treatises

oo
i.s
1. Bahya Bastur Sahit

iv
sh
Manavprakritir
Sambandha Vichar.
RAMAKRISHNA PARAMAHAMSA: 2. Dharmaneeti
Ramakrishna Math
Spreading the ideals of Vedanta.
preaching, philanthropic and
charitable work, all men, women and
children, irrespective of caste, creed,
etc. as equal.
HENRY VIVIAN DEROZIO:
Inspirer of progressive trend, influence of
French revolution, supported women’s
right and education, first nationalist poet
of modern India

WESTERN INDIA:

BALSHASTRI JAMBHEKAR: Father of Marathi


Journalism, attacked orthodoxy.
STARTED NEWSPAPER: Darpan in 1832, as 1st Marathi
Newspaper, Digdarshan in 1840.
Bombay Native General Library and Native
Improvement Society.

Jyotiba Phule: Satyashodhak Samaj in 1873.


Aim: Complete abolition of caste system and socio
economic inequalities, education among women and
lower caste people.
Works: Sarvajanik Satyadharma and Gulamgiri.
Awarded by title of Mahatma.

Pandita Ramabai: Arya Mahila Samaj, Mukti Mission,


Sharda Sadan.
Work areas: Against child marriages, promotion of
girls education and improvement of conditions of
women.

ONLINE | DELHI | AHMEDABAD | BHOPAL | CHANDIGARH | GUWAHATI | HYDERABAD | JAIPUR | JODHPUR | LUCKNOW | PRAYAGRAJ | PUNE | RANCHI | SIKAR 67
SOUTH INDIA:

REFORMER MOVEMENT/ ORGANIZATION ASSOCIATED


SREE NARAYANA GURU Sree Narayana Guru Dharma Paripalana (SNDP) Movement
Aruvippuram movement.

E.V. RAMASWAMY NAICKER Self-Respect Movement

KANDUKURI VEERESALINGAM Hitakarini (Benefactor).

TELANGANA

ANDHRA
PRADESH

KARNATAKA

ONLINE | DELHI | AHMEDABAD | BHOPAL | CHANDIGARH | GUWAHATI | HYDERABAD | JAIPUR | JODHPUR | LUCKNOW | PRAYAGRAJ | PUNE | RANCHI | SIKAR 68
Working Class Movement
Total No. of Questions: 02

Acts/Legislations

Year 2017

Q1. The Trade Disputes Act of 1929 provided for

(a) the participation of workers in the management of industries.


(b) arbitrary powers to the management to quell industrial disputes.
(c) an intervention by the British Court in the event of a trade dispute.
(d) a system of tribunals and a ban on strikes.

ANSWER D

SOURCE: India's struggle for independence, Bipin chandra and Spectrum.

EXPLANATION

Trade Disputes Act (TDA), 1929 made compulsory the appointment of Courts of Inquiry and
Consultation Boards for settling industrial disputes;
made illegal the strikes in public utility services like posts, railways, water and electricity, unless each
individual worker planning to go on strike gave an advance notice of one month to the administration;
forbade trade union activity of coercive or purely political nature and even sympathetic strikes.

Acts/Legislations

Year 2017

Q2. Consider the following statements:


1. The Factories Act, 1881 was passed with a view to fix the wages of industrial workers and to
allow the workers to form trade unions.
2. N. M. Lokhande was a pioneer in organizing the labour movement in British India.
Which of the above statements is/are correct?

(a) 1 only
(b) 2 only
(c) Both 1 and 2
(d) Neither 1 nor 2

ANSWER B

SOURCE: India's struggle for independence, Bipin chandra, Chapter - the Indian working
class and the national movement

ONLINE | DELHI | AHMEDABAD | BHOPAL | CHANDIGARH | GUWAHATI | HYDERABAD | JAIPUR | JODHPUR | LUCKNOW | PRAYAGRAJ | PUNE | RANCHI | SIKAR 69
EXPLANATION

The factories act 1881 primarily dealt with problem of child labour between 7-12 years of age. Hence
statement 1 is not correct.
Narayan Meghaji Lokhande pioneered the labour movement in India and he is remembered for
ameliorating the working conditions of textile mill-hands in the 19th century. He is also knows as the
Father of Trade Union Movement in India. Hence statement 2 is correct.

Additional Information: Working Class Movement Acts and Legislation


The Indian Factory Act, 1881
The working conditions in factories and plantations in nineteenth-century India were
miserable. Ironically, the first-ever demand for regulation of the condition of workers in
factories in India came from the Lancashire textile capitalist lobby. They demanded the
appointment of a commission for investigation into factory conditions. The first commission
was appointed in 1875 although the first Factory Act was not passed before 1881.
The Indian Factory Act, 1881 dealt primarily with the problem of child labour (between 7 and
12 years of age). Its significant provisions were:
employment of children under 7 years of age prohibited,
working hours restricted to 9 hours per day for children,
children to get four holidays in a month,
hazardous machinery to be properly fenced off

The Indian Factory Act, 1891


increased the minimum age (from 7 to 9 years) and the maximum (from 12 to 14 years)
for children,
reduced maximum working hours for children to 7 hours a day,
fixed maximum working hours for women at 11 hours per day with a one-and-a-half
hour interval (working hours for men were left unregulated).
provided weekly holiday for all
But these laws did not apply to British-owned tea and coffee plantations where the
labour was exploited ruthlessly.
The early nationalists, especially the Moderates were indifferent to the labour’s cause.
They believed that labour legislations would affect the competitive edge enjoyed by the
Indian-owned industries. Also, They did not want a division in the movement on the basis
of classes. Hence, did not support the Factory Acts of 1881 and 1891 for these reasons.

ONLINE | DELHI | AHMEDABAD | BHOPAL | CHANDIGARH | GUWAHATI | HYDERABAD | JAIPUR | JODHPUR | LUCKNOW | PRAYAGRAJ | PUNE | RANCHI | SIKAR 70
A PRE-GLIMPSE
of Working Class Movement

MOVEMENT OF THE WORKING CLASS


Phase 1 (1850-1906)
Early nationalists, especially the moderates were usually elites. They did riot give adequate attention
to the labour's cause.
They did not want a division in the movement on the basis of classes Thus did not support the
Factories Act of 1881 and 1891 for these reasons.
Individual Efforts:
(a) Sasipada Banerjee started a workingmens club and newspaper Bharat Shramjeevi in 1870.
(b) N.M. Lokhanday started the newspaper Deenbandhu and set up the Bombay Mill and
Mill hands Association.

Phase 2 (1907-19)
During Swadeshi upsurge: Strikes were organised by Ashwini Coomar Banerjee, Prabhat Kumar Roy
Chaudhuri, Premiosh Bose and Apurba Kumar Ghosh.Subramaniya Siva and Chidambaram Pillai led
strikes in Tuticorin and Tirunelvelli.
During First World War: Discontent among workers due to rise in exports, soaring prices, massive
profiteering opportunities for the industrialists but very low wages for the workers.
Emergence of Gandhi led to a broad-based national movement and mobilisation of the workers.
Gandhi himself fought for the cause of Ahmedabad mill workers in 1918.

Phase 3 (1920-27)
AITUC was founded in 1920. Lala Lajpai Rai, was elected as the first president of AITUC and Dewan
Chaman Lal as the first general secretary.
Gaya session of the Congress (1922) welcomed the formation of the AITUC.
Leaders like C.R.Das, Nehru, Subhash Chandra Bose, C.F. Andrews, J.M. Sengupta, Satyamurthy, V.Y.
Giri and Sarojini Naidu kept close contacts with the AITUC.
Communist Party of India was founded in 1925.
Trade Union Act, 1926 recognised trade unions as legal associations.

Phase 4 (1928-47)
Meerut Conspiracy Case(1929): Communist leaders like Muzaffar Ahmed, S.A. Dange, Joglekar, Philip
Sprat Ben Bradley, Shaukat Usmani and others were arrested. The trial got worldwide publicity but
weakened the working class movement.
Alarmed at the increasing strength of trade union movements, government passed the Public
Safety Ordinance (1929) and the Trade Disputes Act (1929).
After the withdrawal of CDM, young Congress leaders were disillusioned and decided to found CSP
in 1934. CSP continued to work inside the Congress to give it a left leaning.
Congress governments formed in the provinces after the 1937 were generally sympathetic to the
workers demands and gave a fillip to the trade union activity.
During Second world war, communists dissociated themselves from the Quit India Movement. In
the period 1945 to 1947, workers participated actively in the post-War national upsurges like Naval
Ratings.

ONLINE | DELHI | AHMEDABAD | BHOPAL | CHANDIGARH | GUWAHATI | HYDERABAD | JAIPUR | JODHPUR | LUCKNOW | PRAYAGRAJ | PUNE | RANCHI | SIKAR 71
Personality Based Questions
Total No. of Questions: 06

Personality Based Questions, Total No.of Questions: 6

1 4 1
Questions Question Questions

Pre Congress Freedom Struggle Freedom Struggle


Associations Indian Phase Abroad

Pre Congress Associations

Year 2017

Q1. Consider the following pairs:


1. Radhakanta Deb – First President of the British Indian Association
2. Gazulu Lakshminarasu Chetty – Founder of the Madras Mahajana Sabha
3. Surendranath Banerjee – Founder of the Indian Association

Which of the above pairs is/are correctly matched?


(a) 1 only
(b) 1 and 3 only
(c) 2 and 3 only
(d) 1, 2 and 3

ANSWER B

SOURCE: Ncert Class 8th-the Making Of The National Movement: 1870s-1947.

EXPLANATION
The British Indian Association was established on 31 October 1851. The first committee of the
association was composed of : Raja Radhakanta Deb – President, Raja Kalikrishna Deb –
Vice-President, Debendranath Tagore – secretary, Digambar Mitra – Asst Secretary. Hence pair 1 is
correctly matched.

In May 1884, M. Veeraraghavachariar, G. Subramania Iyer and P. Anandacharlu


established the Madras Mahajana Sabha. Hence pair 2 is not correctly matched.

Gazulu Lakshminarasu Chetty was an Indian merchant, Indian independence activist and political
activist who founded the Madras Native Association.
Indian Association was the first declared nationalist organization founded in
British India by Surendranath Banerjee and Anand Mohan Bose in 1876. Hence pair 3 is correctly
matched.

ONLINE | DELHI | AHMEDABAD | BHOPAL | CHANDIGARH | GUWAHATI | HYDERABAD | JAIPUR | JODHPUR | LUCKNOW | PRAYAGRAJ | PUNE | RANCHI | SIKAR 72
A PRE-GLIMPSE
TO EARLY POLITICAL ORGANIZATIONS
BEFORE INC

Organizations in Bengal

ORGANIZATION YEAR OF FOUNDATION FOUNDER

Bangabhasha Associates of Raja


Prakashika Sabha 1836 Rammohan Roy
Zamindari Association/
1838 Dwarakanath Tagore
Landholders' Society

The Bengal British George Thompson


1843
India Society
By merging Landholders'
British Indian Association 1851 Society and the Bengal
British India Society

The East India Association 1866 Dadabhai Naroji

The Indian League 1875 Sisir Kumar Ghosh

Indian Association of Calcutta Surendranath Banerjee


1876
and Ananda Mohan Bose

Organizations in Bombay

ORGANIZATION YEAR OF FOUNDATION FOUNDER

On the lines of British


The Bombay Association 1852
India Association of Calcutta

The Poona Sarvajanik Sabha 1867 Mahadeo Govind Ranade


The Bombay Badruddin Tyabji, Pherozshah
Presidency Association 1885
Mehta and K.T. Telang

Organizations in Madras

ORGANIZATION YEAR OF FOUNDATION FOUNDER

A branch of British Indian


Madras Native Association 1852 association of Calcutta

M. Viraraghavachari,
B. Subramaniya Aiyer and
Madras Mahajan Sabha 1884
P Ananda Charlu

ONLINE | DELHI | AHMEDABAD | BHOPAL | CHANDIGARH | GUWAHATI | HYDERABAD | JAIPUR | JODHPUR | LUCKNOW | PRAYAGRAJ | PUNE | RANCHI | SIKAR 73
Freedom Struggle Indian Phase

Year 2018

Q.2 He wrote biographies of Mazzini, Garibaldi, Shivaji and Shrikrishna; stayed in America for
some time; and was also elected to the Central Assembly. He was

(a) Aurobindo Ghosh


(b) Bipin Chandra Pal
(c) Lal Lajpat Rai
(d) Motilal Nehru

ANSWER C

SOURCE: Political Thinkers of Modern India:Lala Lajpat Rai by Verinder Grover

EXPLANATION

Lala Lajpat Rai was gifted with a perceptive mind, he was a prolific writer and authored several
works like – “Unhappy India”, “Young India: An Interpretation”, “History of Arya Samaj”,
“England’s Debt to India” and a series of popular biographies. His biographies of Mazzini, Garibaldi
and Shivaji were published in 1896 and those of Dayanand and Shri Krishna in 1898.
His purpose in selecting Mazzini and Garibaldi was to infuse patriotic sentiment in the youth of
Punjab, who had no access to books in English. He wanted his countrymen to become acquainted
with the teachings of Italian leaders who had so impressed his own mind.
He joined Swaraj Party in 1926 and was elected its Deputy Leader in the Central Legislative
Assembly. He later resigned from the Swaraj Party in August 1926.

Freedom Struggle Indian Phase

Year 2019

Q.3 Consider the following pairs:

Movement/Organization Leaders
1. All India Anti-Untouchability League : Mahatma Gandhi
2. All India Kisan Sabha : Swami Sahajanand Saraswati
3. Self-Respect Movement : E. V. Ramaswami Naicker

Which of the pairs given above is/are correctly matched?

(a) 1 only
(b) 1 and 2 only
(c) 2 and 3 only
(d) 1, 2 and 3

ANSWER D

ONLINE | DELHI | AHMEDABAD | BHOPAL | CHANDIGARH | GUWAHATI | HYDERABAD | JAIPUR | JODHPUR | LUCKNOW | PRAYAGRAJ | PUNE | RANCHI | SIKAR 74
SOURCE: Spectrum A Brief History Of Modern India

EXPLANATION

In order to better organize the campaign for the amelioration of the untouchables' condition,
Gandhi set up a new body in October 1932. It was first named the All India Anti-Untouchability
League and later renamed the Harijan Sevak Sangh. Hence pair 1 is correctly matched.
Gandhi started publishing a weekly journal called "Harijan" on 11 February 1933 from
Yerwada Jail during British rule.
All India Kisan Congress Sabha, 1936 was founded in Lucknow in April 1936 with Swami
Sahjanand Saraswati as the President and N.G. Ranga as the general secretary. Hence, pair 2
is correctly matched.
E. V. Ramaswami Naicker organized the "Self Respect Movement", designed as Dravidian Uplift,
seeking to expose Brahminical tyranny and the deceptive methods by which they controlled all
spheres of Hindu life. Hence, pair 3 is correctly matched.

Freedom Struggle Indian Phase

Year 2019

Q.4 With reference to Indian National Movement, consider the following pairs:

Person Position held


1. Sir Tej Bahadur Sapru President, All India Liberal Federation
2. K. C. Neogy Member, The Constituent Assembly
3. P. C. Joshi General Secretary, Communist Party of India
Which of the pairs given above is/are correctly matched?

(a) 1 only
(b) 1 and 2 only
(c) 3 only
(d) 1, 2 and 3

ANSWER D

SOURCE: Spectrum A Brief History Of Modern India

EXPLANATION
Tej Bahadur Sapru was a prominent Indian freedom fighter, lawyer and politician. When the
Montagu report of 1918 was made public, there was a divide in the Congress over it. The
moderates welcomed it while the extremists opposed it. This led to a schism in the Congress with
moderate leaders forming the "Indian National Liberal Federation" in 1919. The party (INLF) was
founded by Surendra Nath Banarjea and some of its prominent leaders were Tej Bahadur
Sapru, V. S. Srinivasa Sastri and M. R. Jayakar.
KC Neogy, was an Indian politician from West Bengal. He was a member of the Constituent
Assembly of India, member of the first Cabinet of independent India and the chairman of the
first Finance Commission of India.
Puran Chand Joshi, one of the early leaders of the communist movement in India. He was the
first general secretary of the Communist Party of India from 1935–47.
Hence all the pairs given are correctly matched.

ONLINE | DELHI | AHMEDABAD | BHOPAL | CHANDIGARH | GUWAHATI | HYDERABAD | JAIPUR | JODHPUR | LUCKNOW | PRAYAGRAJ | PUNE | RANCHI | SIKAR 75
Freedom Struggle India

Year 2021

Q.5 In the context of Colonial India, Shah Nawaz Khan, Prem Kumar Sehgal and Gurbaksh Singh
Dhillon are remembered as

(a) leaders of Swadeshi and Boycott Movement


(b) members of the Interim Government in 1946
(c) members of the Drafting Committee in the Constituent Assembly
(d) officers of the Indian National Army

ANSWER D

SOURCE: Old NCERT, Class XII, Modern India, Page No.: 266

EXPLANATION

The British had initially decided to hold public trials of several hundreds of INA prisoners besides
dismissing them from service and detaining without trial around 7,000 of them.
They compounded the folly by holding the first trial at the Red Fort in Delhi in November 1945 and
putting on dock together three INA members - a Hindu, Prem Kumar Sehgal, a Muslim, Shah
Nawaz Khan, and a Sikh, Gurbaksh Singh Dhillon.
Hence option (d) is the correct answer.

Additional Information: INA Trials


INA soldiers were captured. They were taken to court martial at the Red Fort of Delhi.
Some of the prominent examples: Colonel Prem Sahgal, Gurubaksh Singh Dhillon and Major
General Shah Nawaz Khan.
Senior leaders like Bhulabhai Desai, Tej Bahadur Sapru, K.N. Katju, J. Nehru and Asaf Ali
defended PoWs.
The cause was supported by many organizations with varying degree like Congress, ML,
Communist Party, Unionists, Akalis, Justice Party, RSS, Hindu Mahasabha etc

ONLINE | DELHI | AHMEDABAD | BHOPAL | CHANDIGARH | GUWAHATI | HYDERABAD | JAIPUR | JODHPUR | LUCKNOW | PRAYAGRAJ | PUNE | RANCHI | SIKAR 76
Freedom Struggle Abroad

Year 2022

Q6. Consider the following freedom fighters:


1. Barindra Kumar Ghosh
2. Jogesh Chandra Chatterjee
3. Rash Behari Bose

Who of the above was/were actively associated with the Ghadar Party?

(a) 1 and 2
(b) 2 only
(c) 1 and 3
(d) 3 only

ANSWER D

SOURCE: Spectrum A Brief History Of Modern India

EXPLANATION
Rash Behari Bose was one of the most prominent leaders of the Indian National Movement,
especially during the extremist phase. He was one of the masterminds behind the Delhi conspiracy
case, participated in the Ghadar movement, and established the Indian Independence league
while in Japan.
Hindustan Republican Association/Army or HRA (later renamed Hindustan Socialist Republican
Association or HSRA) was founded in October 1924 in Kanpur by Ramprasad Bismil, Jogesh Chandra
Chatterjee and Sachin Sanyal.

The first revolutionary groups in Bengal were organised in 1902 in Midnapore (under
Jnanendranath Basu) and in Calcutta (the Anushilan Samiti founded by Promotha Mitter, and
including Jatindranath Banerjee, Barindra Kumar Ghosh and others.)

ONLINE | DELHI | AHMEDABAD | BHOPAL | CHANDIGARH | GUWAHATI | HYDERABAD | JAIPUR | JODHPUR | LUCKNOW | PRAYAGRAJ | PUNE | RANCHI | SIKAR 77
A PRE-GLIMPSE
GADAR MOVEMENT

GADAR MOVEMENT

It was initiated by NRIS especially in Canada and the Western Coast of the USA.
The first meeting was held in the house of Kashiram attended by Bhai Paramanad, Harnam
Singh Tundilat, Sohan Singh Bhakna.
In the meeting, it was decided to establish Yugantar Ashram in San Francisco & also decided
to release a weekly publication(newspaper/magazine) the Gadar to expose the exploitative
nature of the British rule

Activities of the Gadarites:


They tried to spread anti-British sentiment and unite the people of India against the British.
They incited the Indian army to revolt.
They tried to enlist the help of Germany.
They supported provincial temporary Govt in Afghanistan created by Mahindra Pratap Singh
and Barkatullah.

Canada & USA, many organizations and newspapers were


set up to voice against the British

Swadesh Sevak United India Free Hindustan Circular-i-Azadi Hindi Association


at Vancouver by House at Seatle, newspaper by by Ramnath Puri of Portland set
GD Kumar USA by Tarak- Tarak Nath Das up by Lala
nath & GD Kumar Hardayal in 1913

ONLINE | DELHI | AHMEDABAD | BHOPAL | CHANDIGARH | GUWAHATI | HYDERABAD | JAIPUR | JODHPUR | LUCKNOW | PRAYAGRAJ | PUNE | RANCHI | SIKAR 78
Komagata Maru incident

A ship named Komagata Maru was commissioned to carry some Indians to Canada.
It was carrying 376 passengers who were immigrants from Punjab, India. Of these, only 24 were
granted admittance in Canada.
At that time, Canada had laws restricting entry of migrants of Asian origin
The rest were forced to return back to India
Finally, when they reached the Coast of Budge-Budge(In Bengal) they clashed with the British
authorities which led to death of many Indians.
In this scenario, Indians decided to revolt against the British under the leadership of Rash Bihari
Bose.

ONLINE | DELHI | AHMEDABAD | BHOPAL | CHANDIGARH | GUWAHATI | HYDERABAD | JAIPUR | JODHPUR | LUCKNOW | PRAYAGRAJ | PUNE | RANCHI | SIKAR 79
Chronology
Total No. of Questions: 02

STUDENTS NOTE
Motivation: To check the basic knowledge of all the important historical
events.

Freedom Struggle Phase

Year 2017

Q1 With reference to Indian freedom struggle, consider the following events:


1. Mutiny in Royal Indian Navy
2. Quit India Movement launched
3. Second Round Table Conference
What is the correct chronological sequence of the above events?

(a) 1-2-3
(b) 2-1-3
(c) 3-2-1
(d) 3-1-2

ANSWER C

SOURCE: NCERT chapter 13 class 12 themes in indian history-3

EXPLANATION
Second Round Table Conference took place on 7th September 1931.
The Quit India Movement, also known as the August Movement was a Civil Disobedience Movement
launched by Gandhi JI on 8th August 1942.
Royal Indian Navy (RIN) mutiny was a rebellion launched on February 18, 1946, by seamen on the
HMIS Talwar. Hence correct chronological sequence of the above events are 3-2-1.

Miscellaneous

Year 2018

Q2. Which among the following events happened earliest?


(a) Swami dayanand established Arya Samaj.
(b) Dinabandhu Mitra wrote Neeldarpan.
(c) Bankim Chandra Chattopadhyay wrote Anandmath.
(d) Satyendranath Tagore became the first Indian to succeed in the Indian Civil Services

ANSWER B

SOURCE: Spectrum A Brief History Of Modern India

ONLINE | DELHI | AHMEDABAD | BHOPAL | CHANDIGARH | GUWAHATI | HYDERABAD | JAIPUR | JODHPUR | LUCKNOW | PRAYAGRAJ | PUNE | RANCHI | SIKAR 80
EXPLANATION
Swami Dayanand established Arya Samaj - 1875
Dinabandhu Mitra wrote Neeldarpan - 1858-59
Bankim Chandra Chattopadhyay wrote Anandmath - 1882
Satyendranath Tagore became the first Indian to succeed in the Indian Civil Services Examination
-1863
Hence option (b) is the correct answer.

ONLINE | DELHI | AHMEDABAD | BHOPAL | CHANDIGARH | GUWAHATI | HYDERABAD | JAIPUR | JODHPUR | LUCKNOW | PRAYAGRAJ | PUNE | RANCHI | SIKAR 81
Miscellaneous
Total No. of Questions: 03

Terminologies/ Important Places/ Personality Based, Total No.of Questions: 3

1 1 1
Question Question Question

Terminologies Important Personality


Places Based
Terminologies

Year 2020

Q1. With reference to the history of India, consider the following pairs:
1. Aurang - In-charge of treasury of the State
2. Banian -Indian agent of the East India Company
3. Mirasidar - Designated revenue payer to the State
Which of the pairs given above is/are correctly matched?

(a) 1 and 2 only


(b) 2 and 3 only
(c) 3 only
(d) 1, 2 and 3

ANSWER B

SOURCE: https://ncert.nic.in/ncerts/l/hess201.pdf
OUR PAST-III, chapter-WEAVERS, IRON SMELTERS AND FACTORY OWNERS

ONLINE | DELHI | AHMEDABAD | BHOPAL | CHANDIGARH | GUWAHATI | HYDERABAD | JAIPUR | JODHPUR | LUCKNOW | PRAYAGRAJ | PUNE | RANCHI | SIKAR 82
EXPLANATION

Aurang – A Persian term for a warehouse –a place where goods are


collected before being sold; also refers to a workshop. Hence pair 1 is
not correctly matched.
Banian: In the 18th and 19th centuries, European merchants opened
and deepened trade routes throughout Asia, Africa, and Latin
America. However, in these markets they faced considerable
challenges due to linguistic and cultural barriers. This led to difficulties
in integrating into indigenous commercial and political systems,
which restricted their operations.
The use of intermediaries with expertise of the local markets and
languages rapidly proliferated. In South-East Asia, this was known
as the ‘comprador’ system, whilst in the Anglo-Indian trade
individuals carrying out these functions were known as ‘banians’.
These intermediaries fulfilled various internal and external roles for
trading companies including, managing treasury functions, securing
credit, and acting as brokers in the local markets. A contemporary
described the banian as an individual, “By whom the English
gentlemen in general conduct all their business. He is interpreter,
head book-keeper, head secretary, head broker, the supplier of cash
and cash-keeper, and in general also secret-keeper.
Hence pair 2 is correctly matched.
Mirasidars: Mirasdars were hereditary peasant proprietors who cultivated their own fields and
paid land tax at fixed rates to the state. The Mirasdar belonged to the village and could not be
dispossessed of it so long as he continued to pay the rent. He could sell and transfer his fields and
had the right to sit in the village council. All the land which did not belong to the Mirasdar belonged to
government or those to whom government assigned it. His lands were measured out and classified,
and the standard demand on them fixed. If, however, the rains failed, if the village suffered from war
or pestilence or if a family calamity intervened, he could seek remission.
Hence pair 3 is correctly matched.

Additional Information:
Under the ryotwari settlement system, the government recognized mirasidars as the sole
proprietors of land, dismissing tenants' rights completely. Only in villages where no
mirasidar system existed were those villagers holding permanent occupancy rights
recognized as landholders responsible for the payment of land revenue.

ONLINE | DELHI | AHMEDABAD | BHOPAL | CHANDIGARH | GUWAHATI | HYDERABAD | JAIPUR | JODHPUR | LUCKNOW | PRAYAGRAJ | PUNE | RANCHI | SIKAR 83
Important Places

Year 2021

Q2
With reference to Madanapalle of Andhra Pradesh, which one of the following
statements is correct?
(a) Pingali Venkayya designed the tricolour Indian National Flag here.
(b) Pattabhi Sitaramaiah led the Quit India Movement of Andhra region from here.
(c) Rabindranath Tagore translated the National Anthem from Bengali to English here.
(d) Madame Blavatsky and Colonel Olcott set up headquarters of Theosophical Society
first here
Which of the pairs given above is/are correctly matched?

ANSWER C

SOURCE: Class 6th NCERT: Social & Political Life-I, Unit 1.

The song Jana-gana-mana, composed originally in Bangla by Rabindranath Tagore, was


adopted in its Hindi version by the Constituent Assembly as the National Anthem of India on
January 24, 1950.
It was first sung on December 27, 1911 at the Calcutta Session of the Indian National Congress.
The complete song consists of five stanzas.
It was at Madanapalle (Chittoor), in February 1919, that Tagore translated his Bengali poem
‘Jana Gana Mana’ into English as ‘Morning Song of India’.
It was given tune by Margaret Cousins.

ONLINE | DELHI | AHMEDABAD | BHOPAL | CHANDIGARH | GUWAHATI | HYDERABAD | JAIPUR | JODHPUR | LUCKNOW | PRAYAGRAJ | PUNE | RANCHI | SIKAR 84
Important Personalities

Year 2023

Q3. With reference to the Indian History, Alexander Rea, A. H. Longhurst, Robert
Sewell, James Burgess and Walter Elliot were associated with
(a) archaeological excavations
(b) establishment of English Press in Colonial India
(c) establishment of Churches in Princely States
(d) construction of railways in Colonial India

ANSWER A

SOURCE: Walter Elliot: NCERT: Themes in Indian History Part 1 - Page No.:98

EXPLANATION

Alexander Rea, A. H. Longhurst, Robert Sewell, James Burgess and Walter Elliot were associated
with archaeological excavations.
Alexander Rea was a British archaeologist who worked mainly in South British India. He is known for
unearthing a sarcophagus from the hillocks of Pallavaram in Tamil Nadu.
Albert Henry Longhurst was a British archaeologist and art historian, working in India and Ceylon.
Longhurst was the brother-in-law of Sir John Marshall, the Director-General of the Archaeology
Survey of India (1902-1928). In October 1913 he was appointed the Superintendent of the Southern
Circle, Archaeological Survey of India.
From 1927 to 1931 he was in charge of the systematic digging of Nagarjunakonda. Longhurst served
as the Archaeological Commissioner, Archaeological Survey of Ceylon between 1934 and 1940,
mainly working at Polonnaruwa but also at Anuradhapura and Sigiriya, concentrating more on
conservation/restoration rather than excavation.
Robert Sewell worked in the civil service of the Madras Presidency during the period of colonial rule in
India. He was Keeper of the Madras Record Office. Sewell's specialism was the Vijayanagara Empire,
about which he authored A Forgotten Empire Vijayanagar: A Contribution to the History of India
(1900). Sewell undertook archaeological work, including at the Buddhist stupa at Amaravati, which
had already been largely destroyed prior to his arrival. The site had previously been surveyed by
Colin Mackenzie and Walter Elliot.
In 1854, Walter Elliot, the commissioner of Guntur (Andhra Pradesh), visited Amaravati and
collected several sculpture panels and took them away to Madras. (These came to be called the
Elliot marbles after him.) He also discovered the remains of the western gateway and came to the
conclusion that the structure at Amaravati was one of the largest and most magnificent Buddhist
stupas ever built.

ONLINE | DELHI | AHMEDABAD | BHOPAL | CHANDIGARH | GUWAHATI | HYDERABAD | JAIPUR | JODHPUR | LUCKNOW | PRAYAGRAJ | PUNE | RANCHI | SIKAR 85
ISHITA KISHORE GARIMA LOHIA UMA HARATHI N

CIVIL SERVICES
EXAMINATION 2020

HEAD OFFICE MUKHERJEE NAGAR CENTRE FOR DETAILED ENQUIRY


Apsara Arcade, 1/8-B, Plot No. 857, Ground Floor, Please Call:
1st Floor, Near Gate-6, Mukherjee Nagar, Opposite +91 8468022022,
Karol Bagh Metro Punjab & Sindh Bank, Mukherjee +91 9019066066
DELHI Station, Delhi Nagar, Delhi

AHMEDABAD BHOPAL CHANDIGARH GUWAHATI HYDERABAD JAIPUR JODHPUR LUCKNOW PRAYAGRAJ PUNE RANCHI SIKAR

You might also like